You are on page 1of 42

VISIONIAS

www.visionias.in
ANSWERS & EXPLANATIONS
GENERAL STUDIES (P) TEST – 2975 (2020)

Q 1.A
• Religious reform began among the Parsis in Mumbai in the middle of the 19th century. In 1851, the
Rehnumai Mazdayasan Sabha or Religious Reform Association was founded by Nauroji Furdonji,
Dadabhai Naoroji, S.S. Bengalee and others (Pherozeshah Mehta was not associated with it).
• They started a journal Rast Goftar, for the purpose of social-religious reforms among the Parsis.
• They also played an important role in the spread of education, specially among girls. They campaigned
against the entrenched orthodoxy in the religious field and initiated the modernization of Parsi social
customs regarding the education of girls marriage and the social position of women in general.
• In course of time, the Parsis became socially the most westernized section of Indian society.

Q 2.D
• The Lunar Reconnaissance Orbiter is a NASA robotic spacecraft currently orbiting the Moon in an
eccentric polar mapping orbit. Data collected by LRO has been described as essential for planning
NASA's future human and robotic missions to the Moon.
• Recently, the Lunar Reconnaissance Orbiter imaged the area of the targeted Chandrayaan-2
Vikram landing site on October 14, but did not observe any evidence of the lander. The camera team
carefully examined the images and employed the change detection technique -- using a ratio of an image
captured prior to the landing attempt to the one acquired on October 14. This approach is used for finding
new meteorite impacts on the Moon that also helped locate the recent Beresheet, a small robotic Moon
lander and lunar probe operated by the Israel Aerospace Industries. On April 11 this year, Beresheet
crashed on the Moon during landing.

Q 3.D
• Statement 1 is not correct: The term Depressed Classes was first used by Britishers. The first attempt
for a separate enumeration of the Depressed Classes by any of the Census Commissioners was made in
1911.
• The Government of India Act 1935 replaced the term ‘depressed classes’ with ‘Scheduled Castes’.
Accordingly, separate lists of Scheduled Castes were notified for various provinces in 1936. The Act
also defined races or tribes which appear to His Majesty-in-Council to correspond to the classes of
persons formerly known as the depressed classes as His Majesty-in-Council may specify’.
• Statement 2 is not correct: The legal origin of Reservation Policy in India began with the Government
of India Act, 1919 which came during the turbulent period of World War I. It provided the depressed
classes with 13 seats in provincial legislative council to be filled on the basis of nomination. Accordingly,
Madras was to send 5 members; Bihar, Orissa and Central Provinces 2 each; and, Assam and Bengal one
each.
Q 4.D
• Recently, the Asiatic Society of Mumbai, which was started by and for white European men in the
early 19th century, elected the first woman president in the 215 years of its existence. Prof Vispi
Balaporia, visiting faculty at Mumbai’s Jai Hind College, an institution where she was earlier Vice-
Principal and Head of the Department of English, will head the institution that is a treasure house of
remarkable historical artefacts.
• The Asiatic Society of Mumbai forms part of the network of institutions created by the British to
generate, systematize and disseminate knowledge of India and East: a vast and cumulative body of

1 www.visionias.in ©Vision IAS


information, learning and knowledge which became constituted into the field of Indology. Hence
statement 1 is not correct.
• In 1841, the Society started publishing a journal titled Journal of the Bombay Branch of the Royal Asiatic
Society (RAS) which continues to be published under the name Journal of the Asiatic Society of Mumbai.
That was also the year the Society began admitting Indians as members. In 1954, it became separate from
RAS and renamed itself as The Asiatic Society of Bombay and in 2002 "The Asiatic Society of
Mumbai."
• The business of the Society is managed by office bearers comprising of a President, 4 Vice Presidents, a
Secretary and a Managing Committee of 15 members, all of whom are elected by the General Body. All
are honorary posts. The Governor of Maharashtra is the ex officio Chief Patron of the Society.
• In August 1917 Annie Besant was made the President of the Calcutta Session of the Indian National
Congress. Hence statement 2 is not correct.

Q 5.D
• Statement 3 is not correct: The National Council of Women in India was formed in 1925 as a national
branch of the International Council of Women. However, the National Council of Women did not merge
with the All-India Women's Conference.
• Statement 1 is not correct: The Council was elitist in nature and hence failed to grow and become a
vital national organization. The annual membership fee was Rs.15, it took Rs 500 to become a life
member and Rs.1,000 to become a patron. The members were all women of wealth and position, capable
of affording the expensive travel expected of the Council's leaders and with enough space to house the
Council office. Many women simply could not afford to join this organization nor did they feel
comfortable in the presence of these affluent and titled women.
• Statement 2 is not correct: To a great extent, the works of the Council were determined by the interest of
the individual members. Standing sectional committees were formed to deal with art, labour, legislation,
and the press. The Council primarily looked to the government for the improvements they desired. They
were confident that they understood both the problems of Indian women and the solutions to these
problems and advised the government on welfare issues. They found this work agreeable and their
contacts, established through family, marriage, and social interaction, gave them a credibility that
exceeded their experience or numbers.
• All India Women's Conference
o The All India Women’s Conference (AIWC), founded by Margaret Cousins in 1927, was perhaps the
first women’s organisation with an egalitarian approach. Its first conference was held at Ferguson
College, Pune. Important founding members included Maharani Chimnabai Gaekwad, Rani Sahiba of
Sangli, Sarojini Naidu, Kamla Devi Chattopadhyaya and Lady Dorab Tata.
o Its objectives were to work for a society based on principles of social justice, integrity, equal rights
and opportunities; and to secure for every human being, the essentials of life, not determined by
accident of birth or sex but by planned social distribution.
Q 6.B
• Lord Dalhousie came out to India as the Governor-General in 1848, He was from the beginning
determined to extend direct British rule over as large an area as possible.
• The chief instrument through which Lord Dalhousie implemented his policy of annexation was the
Doctrine of Lapse. Under this Doctrine, when the ruler of a protected state died without a natural
heir, his state was not to pass to an adopted heir as sanctioned by the age-old tradition of the country.
Instead, it was to be annexed to the British dominions unless the adoption had been clearly approved
earlier by the British authorities. Many states, including Satara in 1848 and Nagpur and Jhansi in 1854,
were annexed by applying this doctrine.
• Dalhousie also refused to recognise the titles of many ex-rulers or to pay their pensions. Thus, the titles of
the Nawabs of Carnatic and of Surat and the Raja of Tanjore were extinguished. Similarly, after the death
of the ex-Peshwa Baji Rao II, who had been made the Raja of Bithur, Dalhousie refused to extend his pay
or pension to his adopted son, Nana Saheb.
• Lord Dalhousie was keen on annexing the kingdom of Avadh. But the task presented certain difficulties.
For one, the Nawabs of Avadh had been British allies since the Battle of Buxar. Moreover, they had been
most obedient to the British over the years. The Nawab of Avadh had many heirs and could not
therefore be covered by the Doctrine of Lapse. Some other pretext had to be found for depriving him of
his dominions. Finally, Lord Dalhousie hit upon the idea of alleviating the plight of the people of Avadh.

2 www.visionias.in ©Vision IAS


https://telegram.me/pdf4exams/ https://telegram.me/pdf4exams/
Nawab Wajid Ali Shah was accused of having misgoverned his state and of refusing to introduce reforms.
His state was therefore annexed in 1856.
• Hence, only statement 2 is correct.

Q 7.C
• Bharat Bill Payment System (BBPS)
o It is a Reserve Bank of India (RBI) conceptualized system driven by National Payments Corporation
of India (NPCI) that offers interoperable and accessible bill payment service to consumers via digital
(bank channels) as well as through a network of agents & bank branches. Hence statement 1 is
correct.
o BBPS facilitates a plethora of payment modes through which consumers can make a bill payment.
The payment modes options at BBPS are Cash, Cards (Credit, Debit & Prepaid), IMPS, UPI,
AEPS, Internet Banking & UPI, Wallets. Hence statement 2 is correct.
o BBPS covers repetitive payments for everyday utility services such as electricity, water, gas, telecom
(mobile postpaid, landline postpaid, broadband) and Direct-to-Home (DTH). Gradually, the scope
would be expanded to include other types of repetitive payments like insurance premiums, mutual
funds, school fees, EMI"s, municipal taxes etc.
o For an electronic transaction done at the customer"s own bank, there are nil charges. But for a
transaction done at a physical outlet or bank-branches convenience fee would be levied accordingly.

Q 8.A
• Landholders’ Society also known as The Zamindari Association is considered to be the first political
association of modern India. It was formally launched in Calcutta in March 1838. It was founded to
safeguard the interests of the landlords. Although limited in its objectives, the Landholders’
Society marked the beginning of an organized political activity and the use of methods of
constitutional agitation tor the redressal of grievances. Hence statement 1 is correct.
• Raja Radhakant dev, Dwarkanath tagore, Prasanna Kumar Tagore, Rajkamal Sen and Bhabani Charan
Mitra were its founding members.
• In 1851, both the Landholders’ Society and the Bengal British India Society merged into the British
Indian Association. The President of the first committee of the association was Raja Radhakanta Deb,
while Debendranath Tagore was its secretary. The newspaper of this society was “Hindu patriot”, whose
editor was Harish Chandra Mukherjee, which adopted a strongly critical political tone.
• Dwarkanath Tagore was associated with founding of Landholders’ Society and not with the Indian
Association. Hence statement 2 is not correct.
• The Indian League was started in 1875 by Sisir Kumar Ghosh with the object of “stimulating the
sense of nationalism amongst the people” and of encouraging political education.
• The Indian Association of Calcutta superseded the Indian League and was founded in 1876 by
younger nationalists of Bengal, led by Surendranath Banerjea and Ananda Mohan Bose, who were
getting discontented with the conservative and pro-landlord policies of the British Indian Association. It
was the most important of pre- Congress associations and aimed to create a strong public opinion on
political questions, and unify Indian people on a common political programme.

Q 9.B
• Chittaranjan Das (also known as C.R. Das), whose life is a landmark in the history of India's struggle for
freedom, was endearingly called 'Deshbandhu' (friend of the country). He was a lawyer by profession.
• C.R. Das presided over its third and fourth sessions of the All India Trade Union Congress
(AITUC). Hence option 2 is correct.
• C.R. Das played a major role in promoting the Non-cooperation movement and he gave up his practice of
law as part of a boycott of law courts under the movement.
• He, along with Motilal Nehru was instrumental in the formation of the Congress-Khilafat Swaraj
Party better known later as the Swaraj Party. C.R. Das was the President and Motilal Nehru one of the
Secretaries of the new party. Hence option 1 is correct.
• C.R. Das passed away on 16 June 1925 at Darjeeling at the age of 55 while the Civil Disobedience
Movement was launched in 1930. Hence option 3 is not correct.

3 www.visionias.in ©Vision IAS


https://telegram.me/pdf4exams/ https://telegram.me/pdf4exams/

Join Telegram groups


To Boost Your Preparation
PDF4Exams One stop solution for study
Click Here materials of all competitive exams

The Hindu Zone Official


Newspapers & study Click Here
materials

TestSeries4Exam Aall paid test series


Click Here availabble without any cost

All e-Magazines
Estore
in your hand Click Here
Hindi Books
All study materials
Click Here in Hindi

eSandesh (An Indian App)

For More download eSandesh App from play store


https://telegram.me/pdf4exams/ https://telegram.me/pdf4exams/

Q 10.B
• UMMID (Unique Methods of Management and treatment of Inherited Disorders) is an initiative of
the Department of Biotechnology under the Ministry of Science & Technology to tackle inherited
genetic diseases of newborn babies.
• It also undertakes screening of pregnant women for diagnosis for inherited genetic diseases in hospitals at
aspirational districts.
• In India’s urban areas, congenital malformations and genetic disorders are the third most common cause
of mortality in newborns. With a very large population and high birth rate, and consanguineous marriage
favoured in many communities, the prevalence of genetic disorders is high in India.
• UMMID initiative aims to
o establish NIDAN Kendras to provide counselling, prenatal testing and diagnosis, management, and
multidisciplinary care in Government Hospitals wherein the influx of patients is more
o establish a training centre for Clinicians, in Clinical Genetics and Genetic Diagnostics to produce
skilled clinicians in Human Genetics.
• It also imparts latest medical genetic education to medical students to prepare them for era for molecular
medicine.
• The program will be implemented at government hospitals so that people who cannot afford expensive
care for genetic disorders will be benefited.
• Hence option (b) is the correct answer.

Q 11.A
• There were two kinds of reform movements in the 19th century in India:
o Reformist: These movements responded with the time and scientific temper of the modern era.
o Revivalist: These movements started reviving ancient Indian traditions and thoughts and believed that
the western thinking ruined Indian culture and ethos.
• Deoband Movement was a revivalist movement started by orthodox section among the Muslim
ulema- Mohammad Qasim Nanotavi and Rasid Ahmed Gangohi. It was established in Deoband in
Saharanpur district (United Provinces) in 1866 to train religious leaders for the Muslim community. It had
a twin objectives of propagating pure teachings of the Quran and Hadis among Muslims and keeping alive
the spirit of jihad against the foreign ruler. It propagated pure teachings of Quran and Hadis among
Muslims. On the political front, the Deoband School welcomed formation of the Indian National Congress
and in 1888 issued a fatiua (religious decree) against Syed Ahmed Khan’s organisations, the United
Patriotic Association and the Mohammed Anglo-Oriental Association. Hence option 2 is correct.
• The Ahmadiya movement was a reformist movement founded by Mirza Ghulam Ahamad of
Qadiyan (1839-1908) in 1889, who began his work as a defender of Islam against the polemics of the
Arya Samaj and the Christian missionaries. In 1889, he claimed to be Masiha and Mahdi and later also
to be an incarnation of the Hindu god Krishna and Jesus, returned to earth. The Ahmadiya movement
based itself, like the Brahmo Samaj, on the principles of at universal religion of all humanity. Ghulam
Ahmad was greatly influenced by western liberalism theosophy, and the religious reform movements of
the Hindus. The Ahmadiyas opposed Jihad or sacred war against non-Muslims and stressed fraternal
relations among all people. The movement spread western liberal education among Indian Muslims and
started a network of schools and colleges for that purpose.
• Syed Ahmed Khan started the Aligarh movement as a reformist movement. He wanted to reconcile
western scientific education with the teachings of the Quran to be interpreted in the light of contemporary
rationalism and science even though he also held the Quran to be the ultimate.
The Aligarh Movement emerged as a liberal, modern trend among the Muslim intelligentsia based in
Mohammedan Anglo-Oriental College, Aligarh. It aimed at spreading:
o modern education among Indian Muslims without weakening their allegiance to Islam;
o social reforms among Muslims relating to purdah, polygamy, widow remarriage, women's education,
slavery, divorce, etc.
• The ideology of the followers of the Aligarh movement was based on a liberal interpretation of the Quran
and they sought to harmonise Islam with modern liberal culture. They wanted to impart a distinct socio-
cultural identity to Muslims on modern lines. Soon, Aligarh became the centre of religious and cultural
revival of the Muslim community.
• Hence options 1 and 3 are not correct.
4 www.visionias.in ©Vision IAS
https://telegram.me/pdf4exams/ https://telegram.me/pdf4exams/
Q 12.A
• Vishnu Shastri Pandit and Mahadev Govind Ranade founded Widow Remarriage Association
(1861). Hence, pair 1 is not correctly matched.
• Keshub Chandra Sen laid the foundation stone of his new church, the Tabernacle of New Dispensation
and the newly constructed chapel was consecrated in 1869. He introduced into the church the Pilgrimage
to saints, the Flag ceremony, the Arati, the Savitri Vrata and other innovations. He mentions that this New
Dispensation is '...a Divine message sent to India... It comes not to destroy but to fulfil the law and the
prophets. Hence, pair 2 is correctly matched.
• Theosophical Society was founded in the USA in 1875 by a Russian spiritualist Madame H.P. Blavatsky
and an American Col. H.S. Olcott. Its influence spread in India under Annie Beasant in 1893 who played
an important role in India’s struggle for freedom. Hence, pair 3 is not correctly matched.

Q 13.D
• Bal Gangadhar Tilak was an ardent nationalist who helped lay the foundation for India’s independence by
building his own defiance of British rule into a national movement. In addition, he was also a scholar,
mathematician and philosopher.
• Some of his major books include -: The Arctic Home in the Vedas, Chaldean and Indian Vedas and Gita
Rahasya or the Science of Karmayog.
• In 1880s, Tilak founded two daily newspapers, the Kesari and the Mahratta, published in English.
Within two years, Kesari was attracting more readers than any other language newspaper in India. The
editorials vividly portrayed the suffering of the people, and reported on actual events, calling on every
Indian to fight for his rights. The language was intended to arouse, in the most timid reader, a passionate
thirst for freedom.
• Hence, option (d) is the correct answer.

Q 14.B
• Statement 1 is not correct and statement 2 is correct: The Turtle Survival Alliance (TSA) was formed
in 2001 as an International Union for Conservation of Nature (IUCN) partnership for sustainable captive
management of freshwater turtles and tortoises, and initially designated a Task Force of the IUCN
Tortoise and Freshwater Turtle Specialist Group. It is not an inter-governmental body.
• The TSA arose in response to the rampant and unsustainable harvest of Asian turtle populations to supply
Chinese markets, a situation known as the Asian Turtle Crisis.
• Recently, about 70 hatchlings of the rare Black Softshell and Indian Softshell turtles bred in the ponds of
two temples in Assam were released in the Pobitora Wildlife Sanctuary, about 50 km east of
Guwahati. The sanctuary is often called ‘Mini Kaziranga’ because of similar landscape and a sizeable
population of the one-horned rhino.
• The “wild restocking” of the two turtle species was done under a joint programme of the
management committees of two temples — Ugratara in Guwahati and Hayagriv Madhav in Hajo about
35 km north-west — the Assam Forest Department and Turtle Survival Alliance and Help
Earth. Help Earth had last year organised the release of 35 captive-bred turtle hatchlings in the Pobitora
Wildlife Sanctuary.
• This wild restocking programme is important for sustaining turtles in the Brahmaputra river system,
especially the Black Softshell (Nilssonia nigricans) that is considered extinct in the wild. Assam is the
most species-rich State in India in terms of turtle diversity. It is home to 20 species of freshwater
turtles and tortoises out of 29 species found in India. But, 80% of these species are threatened with
extinction.

Q 15.C
• Most significant historical contribution of the moderates was that they offered an economic critique of
colonialism. Dadabhai Naoroji (published Poverty and Un-British Rule in India), justice M.G.
Ranade and R.C. Dutt, a retired JCS officer, who published 'The Economic History of India' were
the main proponents of this critique. The main thrust of this economic nationalism was on Indian
poverty created by the application of the classical economic theory of free trade. This turned India into a
supplier of agricultural raw materials and foodstuffs to and a consumer of manufactured goods from the
mother country. India was thus reduced to the status of a dependent agrarian economy and a field for
British capital investment. Hence, Statement 1 is correct.

5 www.visionias.in ©Vision IAS


https://telegram.me/pdf4exams/ https://telegram.me/pdf4exams/
• It was argued that direct drainage of wealth took place through the payment of home charges,
military charges, and guaranteed interest payments on railway investments. The burden became
even heavier because of the falling exchange rates of rupee in the 1890s and was compounded by
budget deficits, higher taxes, and military expenditure. Hence, Statement 2 is correct.
• In Naoroji's calculation, this huge drainage amounted to about £12 million per year. On average, this
amounted to at least half of the total revenue income of the British Indian government. This directly
impoverished India and stultified the process of capital formation.
• However, it was believed by Nationalists that the key to India's development was industrialization
with Indian capital, while investment of foreign capital meant drainage of wealth through
expatriation of profit. This belief was also motivated by the fact that many of the nationalists were
Capitalists (like Dada Bhai Nairoji) and significant funding of the Indian National Congress came from
the Capitalist class. Hence, Nationalists did not propound the abolition of Capitalism. This theory
came after the Russian Revolution in the 1920s. Democratic Socialism was the economic model proposed
by Mahatma Gandhi. Hence, Statement 3 is not correct.

Q 16.A
• Nadir Shah was attracted to India by the fabulous wealth for which it was always famous. Continual
campaigns had made Persia virtually bankrupt. Money was needed desperately to maintain his mercenary
army. Spoils from India could be a solution. At the same time, the visible weakness of the Mughal Empire
made such spoliation possible.
• He entered Indian territory towards the end of 1738, without meeting with any opposition. For years
the defences of the northwest frontier had been neglected. The danger was not fully recognised till the
enemy had occupied Lahore. Hurried preparations were then made for the defence of Delhi, but the
faction-ridden nobles refused to unite even in sight of the enemy, They could not agree on a plan for
defence or on the commander of the defending forces. Disunity, poor leadership, and mutual jealousies
and distrust led to their defeat.
• The two armies met at Karnal on 13th February 1739 and the invader inflicted a crushing defeat on the
Mughal army. Emperor Muhammad Shah was taken prisoner and Nadir Shah marched on to Delhi.
• A terrible massacre of the citizens of the imperial capital was ordered by Nadir Shah as a reprisal
against the killing of some of his soldiers.
• The greedy invader took possession of the royal treasury and other royal property, levied tribute on the
leading nobles, and plundered the rich of Delhi. His total plunder has been estimated at 70 crores of
rupees. This enabled him to exempt his own Kingdom from taxation for three years. He also carried away
the famous Koh-i-nur diamond and the jewel-studded Peacock Throne of Shahjahan.
• He compelled Muhammad Shah to cede to him all the provinces of the Empire west of the river
Indus.

Q 17.D
• Jahandar Shah’s inglorious reign came to an early end in January 1713 when he was defeated at Agra by
Farrukh Siyar, his nephew.
• Farrukh Siyar owed his victory to the Saiyyid brothers, Abdullah Khan and Husain All Khan
Baraha, who were therefore given the offices of wazir and mir bakshi respectively. The two brothers
soon acquired dominant control over the affairs of the state. Farrukh Siyar lacked the capacity to rule. He
was cowardly, cruel, undependable and faithless. Moreover, he allowed himself to be influenced by
worthless favorites and flatterers.
• In spite of his weaknesses, Farrukh Siyar was not willing to give the Saiyid brothers a free hand but
wanted to exercise personal authority. On the other hand, the Saiyid brothers were convinced that
administration could be carried on properly, the decay of the Empire checked, and their own position
safeguarded only if they wielded real authority and the Emperor merely reigned without ruling. Thus there
ensued a prolonged struggle for power between the Emperor Farrukh Siyar and his wazir and mir bakshi.
From 1713 until 1720, the Saiyid brothers wielded the administrative power of the state.
• The Saiyid brothers adopted the policy of religious tolerance. They believed that India could be ruled
harmoniously only by associating Hindu chiefs and nobles with the Muslim nobles in governing the
country. Again, they sought to conciliate and use the Rajputs, the Marathas, and the Jats in their struggle
against Farrukh Siyar and the rival nobles. They abolished the jizyah immediately after Farrukh
Siyar's accession to the throne. Similarly, the pilgrim tax was abolished from a number of places.

6 www.visionias.in ©Vision IAS


https://telegram.me/pdf4exams/ https://telegram.me/pdf4exams/
• They won over to their side Ajit Singh of Marwar, Jai Singh of Amber, and many other Rajput
princes by giving them high positions of influence in the administration. They made an alliance with
Churaman, the Jat chieftain. In the later years of their administration they reached an agreement with
King Shahu by granting him the swarajya (of Shivaji) and the right to collect the chauth and sardeshmukhi
of the six provinces of the Deccan. In return, Shahu agreed to support them in the Deccan with 15,000
mounted soldiers.
• Hence, only statement 2 is correct.

Q 18.B
• The Indian Merchants' Chamber - 1907
o It was set up in 1907, in the wake of the 'Swadeshi Movement' to represent Indian businesses. It
is a premier Chamber of trade, commerce and industry in India.Its members and its network have
been instrumental in Influencing policy frameworks and changes towards this goal and continually
strengthening sectors that are critical to India's new phase of flourish.It seeks out thought leaders and
Industry spearheads to identify today's needs and catalyse the achievement of tomorrow's vision.
• The Associated Chambers of Commerce and Industry of India (ASSOCHAM) - 1920
o A need was felt in the in the wake of post-war (World War I) scenario for a national business
organisation in response to the integration of economic policy into an all-India pattern, an enlarging
corpus of legislation, the rapid growth of Indian business, a sharper spirit of trade rivalry, as well as
the mobilization of important Indian industrial financial support for political purposes.
o This eventually led to the formation of ASSOCHAM in 1920 with its maiden conference in
Calcutta. It was inaugurated by Lord Chelmsford.
o It was more than a chamber for its members; it served as stimulant to other business groups to close
ranks. Primarily, it inspired the "Indian" sector, which looked at ASSOCHAM as an example of
British business interests lobbying on a national scale and creating the wherewithal for smooth
progress.
• Federation of Indian Chambers of Commerce & Industry (FICCI) - 1927
o Established in 1927, it is one of the largest and oldest apex business organization in India. Its history
is closely interwoven with India's struggle for independence, its industrialization, and its emergence
as one of the most rapidly growing global economies.
o A non-government, not-for-profit organization, FICCI is the voice of India's business and industry.
From influencing policy to encouraging debate, engaging with policymakers and civil society, FICCI
articulates the views and concerns of industry.
o Hence option (b) is the correct answer.

Q 19.D
• In 1829 Raja Rammohun Roy founded a new religious society, the Brahma Sabha, later known as the
Brahmo Samaj, whose purpose was to purify Hinduism and to preach theism or the worship of one
God. The new society was to be based on the twin pillars of reason and the Vedas and Upanishads. It
was also to incorporate the teachings of other religions.
• The Brahmo tradition of Raja Rammohun Roy was carried forward after 1843 by Devendranath Tagore,
who also repudiated the doctrine that the Vedic scriptures were infallible, and after 1866 by Keshub
Chandra Sen. The Brahmo Samaj made an effort to reform Hindu religion by removing abuses, by basing
it on the worship of one God and on the teachings of the Vedas and Upanishads, and by incorporating the
best aspects of modern western thought.
• Most of all it based itself on human reason which was to be the ultimate criterion for deciding what was
worthwhile and what was useless in the past or present religious principles and practices. For that
reason, the Brahmo Samaj denied the need for a priestly class for interpreting religious writings.
Every individual had the right and the capacity to decide with the help of his own intellect what was right
and what was wrong in a religious book or principle.
• The Brahmos were also great social reformers. They actively opposed the Caste system and child-
marriage and supported the general uplift of women, including widow remarriage, and the spread of
modem education to men and women.
• Hence, all the given statements are correct.

7 www.visionias.in ©Vision IAS


https://telegram.me/pdf4exams/ https://telegram.me/pdf4exams/
Q 20.A
• In 1857, Lord Canning passed the General Service Enlistment Act which decreed that all the future
recruits to the Bengal Army would have to give an undertaking to serve anywhere their service might be
required. This became one of the reasons for revolt as for Indians, as serving overseas means crossing the
sea which meant loss of caste status, as those crossing the sea were not taken back to the fold of the caste.
• In the revolt, the lower castes didn't show any active sympathy for the rebellions. Therefore, the
soldiers in the Bombay and Madras army, which had recruits from the lower caste, remained loyal
to the British government. Hence statement 1 is correct.
• The 1857 revolt had a limited territorial outreach. Main areas were Western Bihar, Oudh,
Rohillakhand, Delhi and the territory between Chambal and the Narmada. The regions south of
Narmada were impacted very little. Hence statement 2 is not correct.

Q 21.D
• Recently, a survey was conducted by Nutrition International in collaboration with the All India
Institute of Medical Sciences and the Indian Coalition for the Control of Iodine Deficiency
Disorders (ICCIDD).
• The survey tested the iodine content in samples of cooking salt from households to estimate the coverage
of iodized salt.
• Statement 1 is correct: The study shows that 76.3% of Indian households consumed adequately iodized
salt, which is salt with at least 15 parts per million of iodine. The five worst performers were Tamil Nadu
(61.9%), Andhra Pradesh (63.9%), Rajasthan (65.5%), Odisha (65.8%) and Jharkhand (68.8%).
• Gujarat produces 71% of salt in the country, followed by Rajasthan at 17% and Tamil Nadu at 11%.
The rest of the country accounts for a mere 1% of the salt produced.
• Statement 2 is not correct: As per the survey, Jammu & Kashmir (now a Union Territory) is at the top
with respect to iodized salt consumption at the household level (99.80%), followed by northeastern States
that are doing very well with respect to iodized salt consumption at the household level. Tamil Nadu has
the lowest number of households that consume iodized salt.
• India made fortification of salt with iodine mandatory for direct human consumption in 1992. This was
relaxed in 2000 and then reimposed in 2005. In 2011, the SC, too, mandated universal iodization for the
control of iodine deficiencies.
• Statement 3 is not correct: The Salt Commissioner’s Organization with its headquarters at Jaipur is
headed by the Salt Commissioner, who is assisted by two Assistant Salt Commissioners and other
supporting staff.
• There are four regional offices functioning at Chennai, Mumbai, Ahmedabad, and Jaipur, besides field
offices in all the salt-producing states.
• The Salt Commissioner's Organization is responsible for monitoring production, distribution, and
supply of salt.
• The organization is a nodal agency for monitoring production, distribution, and quality of iodized salt
under the National Iodine deficiency control programme of the Ministry of Health and Family Welfare.
• The other functions of the organization include, price surveillance, promotion of technological
development, maintenance of standards and quality improvement of salt, planning, formulation and
monitoring the execution of development and labor welfare schemes, custody and superintendence of
departmental salt and and other assets, promotion of exports, pre-shipment inspection and rehabilitation of
salt works affected by the natural calamities.

Q 22.A
• 'Galo', 'Wancho' and 'Adi' often seen in news are tribal communities from Arunachal
Pradesh. They have been in the news for various reasons.
o Galo: Recently, members of the Galo community in Arunachal Pradesh were in news for being
able to recollect the names of their ancestors. This is made possible by their system of
naming. The Galos belong to the Tani group inhabiting Assam and Arunachal Pradesh, besides
Tibet. They trace their common origin to a primeval ancestor, Abotani. But unlike the Mising
(Assam), Adi, Apatani, Nyishi and Tagin, the other communities, only the Galos maintain genealogy
through given names. They have a system of prefixing the second syllable of a father’s name to that of
a son, who passes on the suffix in his name to his son. They can thus trace the names of ancestors
from the first syllable or prefix of their names. At about 1.5 lakh people, the Galos are one of the 26

8 www.visionias.in ©Vision IAS


https://telegram.me/pdf4exams/ https://telegram.me/pdf4exams/
major communities of Arunachal Pradesh, and dominate West Siang, Lepa Rada, and Lower Siang
districts. They have a big population in East Siang, Upper Subansiri and Namsai districts too.
o Wancho: Recently, in an attempt to preserve 'Wancho language' spoken in several northeastern
states, linguist student Banwang Losu has developed an independent Wancho script. Wancho,
one of the tribes in Arunachal Pradesh, lacked a script. It is not only confined to Arunachal Pradesh
but is also spoken in Nagaland, Assam and other countries like Myanmar and Bhutan. It is among the
endangered languages of the world. The United Nations had declared 2019 as the year of
Indigenous Languages to draw attention to languages around the world that are in danger of
disappearing. Out of 197 Indian languages on the verge of extinction, 89 languages are reported to be
from Northeast India, with 34 belonging to Arunachal Pradesh alone.
o Adi: The Adi people are one of the most populous groups of indigenous peoples in the Indian
state of Arunachal Pradesh. They are found in the temperate and sub-tropical regions within the
districts of Siang, East Siang, Upper Siang, West Siang, Lower Dibang Valley, Lohit, and Namsai
within Arunachal Pradesh. They have a well-organised village council called ‘Kebang’. Their
traditional dance called ‘Ponung’ is famous in the whole of Arunachal Pradesh. Recently, the chief
minister of Arunachal Pradesh, Pema Khandu, went on a bike ride to promote tourism in the state.
The attempt was towards promoting Arunachal as a dream destination for biking and adventure sports.
He mentioned that the route offers the best picturesque views of the Siang valley and villages of Adi
tribes.

Q 23.B
• Scientists recently unveiled a new species of pterosaur, the plane-sized reptiles that lorded over
primeval skies above T-rex, Triceratops and other dinosaurs of the late Cretaceous.
• With a wingspan of 10 m and weighing 250 kg, Cryodrakon boreas rivals another pterosaur as the
largest flying animal of all time. Its remains were first discovered more than 30 years ago in Alberta. It
is now considered distinct from Quetzalcoatlus. Quetzalcoatlus northropi is a pterosaur known from the
Late Cretaceous of North America and one of the largest known flying animals of all time.
• There are more than 100 known species of pterosaurs. Despite their large size and wide distribution —
across North and South America, Asia, Africa and Europe — only fragmentary remains have been
unearthed, making the new find especially important.

Q 24.A
• Global Fund for AIDS, Tuberculosis and Malaria (GFTAM)
o It is a public-private partnership designed to accelerate the end of AIDS, tuberculosis and malaria as
epidemics. Hence statement 1 is correct.
o It mobilizes and invests more than US$4 billion a year to support programs run by local experts in
more than 100 countries. The money comes 93% from donor governments and 7% from the private
sector and foundations.
o A Country Coordinating Mechanism made up of medical experts, government and civil society are
responsible for the planning and execution of plans to fight the three diseases.
• India vis-a-vis GFTAM
o India shares a sustained partnership with the Global Fund since 2002, both as a recipient and as a
donor. Hence statement 2 is not correct.
o India recently announced a contribution of USD 22 million to the fund for 2020-22 as part of its
commitment towards eliminating these diseases.
o In the current funding cycle (2018-21), the Global Fund has allocated USD 500 million to India. As a
donor, India has contributed USD 46.5 million till 2019, including USD 20 million.

Q 25.D
• The women’s movements began as a social reform movement in the 19th century. The British conquest
and its rule over India brought about a transformation in the Indian economy as well as in society. The
new land revenue, the new economic system, and administrative machinery required a new type of
educated personal which resulted in the establishment of Western educational institutions imparting
modern education. The Indians who were the beneficiaries of the new economic system were attracted to
this and as a result, a new class of intelligentsia evolved in the Indian society.

9 www.visionias.in ©Vision IAS


https://telegram.me/pdf4exams/ https://telegram.me/pdf4exams/
• Spurred by new European ideas of rationalism and progress, the reformers tried to create a new society,
modern yet rooted in Indian tradition. They began a critical appraisal of Indian society in an attempt to
create a new ethos devoid of all overt social aberrations like polytheism, polygamy, casteism, sati, child
marriage, illiteracy, etc. all of which they believed were impediments to the progress of women.
• Statement 1 is not correct: However, the social reform movement did not radically challenge the
existing patriarchal structure of society or question gender relations. They picked up for reform only
those issues which the British were pointing out as evidence of degeneration in the Indian society. Even
the women’s institutions and organizations that sprang up during this period do not reveal the
development of an independent view. As a result, even when women were speaking for themselves they
were speaking only the language of the men, defined by male parameters
• Statement 2 is not correct: Women were seen as passive recipients of a more humanitarian
treatment to be given by Western-educated elite men. There was thus an attempt to reform women
rather than reform the social conditions which opposed them. There were no attempts to alter the power
structure and the men-women relation in society. This was but natural since the change in the status of
women was being sought only within questioning patriarchy itself. The attempt was to create a new Indian
woman, truly Indian and yet sufficiently educated and tutored in the 19th-century values to suit the new
emerging society.
• Statement 3 is not correct: The education for girls was not meant to equip them to be self-sufficient,
independent and emancipated and train them to follow some profession but to be good housewives.
Thus, there was not much emphasis on technical or vocational education but on basic
education. This enabled them to be educated without gaining access to livelihood or gainful employment.
Consequently, education was not meant to equip them to be self-sufficient or independent but to be good
housewives, restricting them to the home and the hearth.

Q 26.D
• Vidyasagar’s contribution to the making of modern India is many-sided. He evolved a new technique of
teaching Sanskrit. He wrote a Bengali primer which is used till this day. By his writings, he helped in the
evolution of a modern prose style in Bengali.
• He opened the gates of the Sanskrit college to non-Brahmin students for he was opposed to the
monopoly of Sanskrit studies that the priestly caste was enjoying at the time. To free Sanskrit studies
from the harmful effects of self-imposed isolation, he introduced the study, of Western thought in the
Sanskrit College. He also helped found a college which is now named after him.
• Above all Vidyasagar is remembered gratefully by his countrymen for his contribution to the uplift of
India’s down-trodden womanhood. He waged a long struggle in favour of widow remarriage. His
humanism was aroused to the full by the sufferings of the Hindu widows. To improve their lot he gave his
all and virtually ruined himself. He raised his powerful voice, backed by the weight of immense
traditional learning, in favour of widow remarriage in 1855. Soon a powerful movement in favour of
widow remarriages was started which continues till this day. Later in the year 1855, a large number of
petitions from Bengal, Madras, Bombay, Nagpur were presented to the Government asking it to pass an
act legalising the remarriage of widows (The Hindu Widows' Remarriage Act, 1856.). This agitation
was successful and such a law was enacted. The first lawful Hindu widow remarriage among the upper
castes in our country was celebrated in Calcutta on 7 December 1856 under the inspiration and
supervision of Vidyasagar. Widows of many other castes in different parts of the country already enjoyed
this right under customary law.
• Hence, all the given statements are correct.

Q 27.B
• Curzon justified greater control over universities in the name of quality and efficiency, but actually sought
to restrict education and to discipline the educated towards loyalty to the Government. With an idea to
bring the Universities under control, which he felt had become hotbeds of revolutionary activities. Lord
Curzon appointed Raleigh Commission under Sir Thomas Raleigh. This commission submitted its
report in 1902 and this followed the introduction of a Bill called Raleigh Bill. The Raleigh Bill when
became an act, it was called Indian Universities Act 1904. Hence statement 3 is correct.
• The Act increased Government's control over the universities. It could veto the regulations passed by
the Senate of the University. It allowed the Government to appoint a majority of the fellows in a

10 www.visionias.in ©Vision IAS


https://telegram.me/pdf4exams/ https://telegram.me/pdf4exams/
university. The Governor General was now empowered to decide a University's territorial limits. Hence
statement 1 is not correct.
• Also, it increased University control over private colleges by laying down stricter conditions for
affiliation and periodical inspection. Here also, the government approval was necessary for grant of
affiliation and disaffiliation of colleges. Hence statement 2 is correct.
• However, for better education and research a grant of Rs. 5 Lakh per year for 5 years was also accepted.

Q 28.C
• Commercial enterprise led the Dutch to undertake voyages to the East. After their arrival in India, the
Dutch founded their first factory in Masulipatnam (in Andhra) in 1605. They went on to establish trading
centres at different parts of India and thus became a threat to the Portuguese. They captured Nagapatam
near Madras (Chennai) from the Portuguese and made it their main stronghold in South India.
• The English were also at this time rising to prominence in the Eastern trade, and this posed a serious
challenge to the commercial interests of the Dutch. A commercial rivalry soon turned into bloody warfare.
After prolonged warfare, both the parties came to a compromise in 1667 by which the British agreed to
withdraw all their claims on Indonesia, and the Dutch retired from India to concentrate on their more
profitable trade in Indonesia. They monopolized the trade in black pepper and spices.
• The Dutch were not much interested in empire-building in India; their concerns were trade. In any case,
their main commercial interest lay in the Spice Islands of Indonesia from where they earned a huge profit
through business.
• The Dutch got drawn into the trade of the Malay Archipelago. Further, in the third Anglo-Dutch War
(1672-74), communications between Surat and the new English settlement of Bombay got cut due to
which three homebound English ships were captured in the Bay of Bengal by the Dutch forces. The
retaliation by the English resulted in the defeat of the DutchThe Battle of Chinsurah (also known as
the Battle of Biderra or Battle of Hoogly) took place near Chinsurah, India on 25 November 1759 ),
which dealt a crushing blow to Dutch ambitions in India.
• Hence, both the statements are correct.

Q 29.B
• Warren Hastings became the Governor of Bengal in 1772 and in 1774. He became the first Governor-
General of Bengal. When he became the Governor of Bengal, he had to face many problems. The ‘Dual
Government’ had caused confusion and disorder in the state. There was no satisfactory arrangement for
the collection of revenue and the revenue income was irregular. The treasury was empty and there was
terrible famine in the country. Warren Hastings carried out a large number of reforms which are:
• In 1781, Warren Hastings set up the Calcutta Madrasah for the study and teaching of Muslim law and
related subjects; and, in 1791, Jonathan Duncan started a Sanskrit College at Varanasi, where he was the
Resident, for the study of Hindu Law and Philosophy. Both these institutions were designed to provide a
regular supply of qualified Indians to help the administration of law in the courts of the Company.
• Administrative Reforms: Warren Hastings put an end to the dual system of government in Bengal which
was established by Robert Clive in 1765. The Company took over the responsibility of the administration
of the province and started to collect the revenue through the agency of its own servants. Mohammad
Reza Khan and Raja Sitab Rai, who were the Deputy Nawabs of Bengal and Bihar, were removed from
their offices. The treasury was shifted from Murshidabad to Calcutta and it became the seat of
government.
• Revenue Reforms: Although the Company had got the Diwani of Bengal, Bihar and Orissa in 1765, the
work of a collection of land revenue had been left in the hands of amils. Warren Hastings decided that the
Company must directly collect the revenue. Consequently, he appointed collectors for revenue
collection and administration, who were to be helped by native officers. Settlement was made for
five years with the highest bidders. To supervise the whole organization, a Board of Revenue was
established at Calcutta. This revenue system was found to be defective and consequently, in 1777, the old
system of bidding for a year was resorted to. He made the account of revenue simple and intelligible and
made many provisions for the protection of ryots.
• Economic Reforms: Warren Hastings prohibited the used of dastaks by the servants of the Company and
thereby added to the revenues of the Company. He abolished a large number of custom houses or
chowkies as they were hampering the growth of trade in the country. He reduced the duties on all goods
except salt, betel nut and tobacco.

11 www.visionias.in ©Vision IAS


https://telegram.me/pdf4exams/ https://telegram.me/pdf4exams/
• Judicial Reforms: Warren Hastings carried out a large number of reforms in the judicial sphere. In 1772,
he established Diwani Adalat (Civil Court) and a Faujdari Adalat (Criminal Court) in each district. The
Diwani Adalat was presided over by the English collector and Faujdari Adalat was presided over by
Indian officers Qazis and Muftis. Two courts of appeal were established at Calcutta. They are the Sadar
Diwani Adalat (the Supreme of Civil Court) and the Sadar Nizamat Adalat (the Supreme Criminal Court)
which heard appeals from the Diwani Adalat and Faujdari Adalat respectively. He also compiled a simple
code about the personal laws of Hindus and Muslims.
• The Treaty of Seringapatam, signed in 1792, ended the Third Anglo-Mysore War. Its signatories
included Lord Cornwallis on behalf of the British East India Company, representatives of the
Nizam of Hyderabad and the Maratha Empire, and Tipu Sultan, the ruler of Mysore.
• Hence statements 1 and 2 are correct and 3 is not correct.

Q 30.A
• At the end of the eighteenth century, Ranjit Singh, chief of the Sukerchakia Misls, rose to prominence. A
strong and courageous soldier, an efficient administrator, and a skillful diplomat, he was a born leader of
men. He captured Lahore in 1799 and Amritsar in 1802. He soon brought all Sikh chiefs west of the Sutlej
under his control and established his own kingdom in Punjab. Later, he conquered Kashmir, Peshawar,
and Multan.
• The old Sikh chiefs were transformed into big zamindars and jagirdars. He did not make any changes in
the system of land revenue promulgated earlier by the Mughals. The amount of land revenue was
calculated on the basis of 50 percent of the gross produce.
• Ranjit Singh built up a powerful, disciplined, and well-equipped army along European lines with the help
of European instructors. His new army was not confined to the Sikhs. He also recruited Gurkhas,
Biharis, Oriyas, Pathans, Dogras, and Punjabi Muslims.
• He set up modern foundries to manufacture cannon at Lahore and employed Muslim gunners to man
them. It is said that he possessed the second-best army in Asia, the first being the army of the English East
India Company.
• Ranjit Singh had great capacity for choosing his ministers and officials. His court was studded with
outstanding men. He was tolerant and liberal in religious matters. He patronized not only Sikh but also
Muslim and Hindu holy men. Many of his important ministers and commanders were Muslims and
Hindus. The most prominent and trusted of his ministers was Fakir Azizuddin, while his finance minister
was Dewan Dina Nath. He was a state based on equal opportunities for all. Political power was not used
for exclusive Sikh benefits.

Q 31.B
• Statement 1 is not correct: Dehradun based Wildlife Institute of India (WII) has launched a mobile
application “Ganga Data Collector” under the “Biodiversity and Ganga Conservation” project initiated by
the National Mission for Clean Ganga (NMCG) of Union Ministry of Jal Shakti. The app will provide
field researchers with a complete data entry solution to monitor the aquatic population in the river.
• Statement 2 is correct: The scientists of WII, Ganga Prahri volunteers and staff of the forest department
in 11 states of the Ganga Basin will use this application on their mobile phones for faster collection of
more authentic and accurate data related to water quality and aquatic life in Ganga. WII will train nearly
550 Ganga Prahris and staff of the forest departments of the states concerned. To conserve the ecological
integrity of the Ganga River, and, reduce the direct dependency of the local communities on the river, the
National Mission for Clean Ganga-Wildlife Institute of India (NMCG-WII) project ‘Biodiversity
Conservation and Ganga Rejuvenation’ is involving members of the local community in the five Ganga
states (Uttarakhand, Uttar Pradesh, Bihar, Jharkhand and West Bengal) as guardians of the river who will
henceforth be known as Ganga Praharis.
• Ganga Praharis are self-motivated and trained volunteers from among the local communities working for
biodiversity conservation and cleanliness of the Ganga River with the ultimate objectives of restoring the
Nirmal and Aviral Dhara. The primary criterion for identification in a candidate is passion and zeal to
serve the Ganga River for maintaining its integrity in terms of its cleanliness and biodiversity value. These
‘Praharis’ have to be above 18 years of age on the date of joining the project, preferably from the Ganga
Riverside villages. The younger school students, too, have been recruited for Bal Ganga Praharis which
will be launched soon.

12 www.visionias.in ©Vision IAS


https://telegram.me/pdf4exams/ https://telegram.me/pdf4exams/
• The Ganga basin covers 11 states including Uttarakhand, Uttar Pradesh, Madhya Pradesh, Rajasthan,
Haryana, Himachal Pradesh, Chhattisgarh, Jharkhand, Bihar, West Bengal and Delhi.

Q 32.C
• Head on Generation (HOG) technology
o It is an electrical power supply system where electrical power for catering hotel load of train, which
includes Train Lighting, Air conditioning, Lighting, fans and other passenger interface requirement is
drawn from the Overhead Electric supply.
o This scheme is widely used power supply system by Railways world over.
• Current Method of power supply in rail coaches
o The railway coaches have been designed to run on the End on Generation (EOG) system with two
power cars employing two DG sets on either end.
o Cost of electricity generation from DG set is very high resulting into increase in diesel consumption
and substantial operating expenditure
o Currently, the cost of power is over Rs. 36 per unit and with HOG it will available at Rs. 6 Per unit.
o The power generator cars also make huge noise and emit pollution fumes.

Q 33.C
• Statement 1 is correct: A vulture restaurant is a site where carrion is deposited for endangered vultures
to feed on where non-toxic, poison-free meat and carcasses are provided. Vulture restaurants can also be
called feeding sites, feeding schemes, and vulture safe zones.
• The survival of vultures in some areas is threatened by a variety of circumstances, including loss of
habitat and diminishing food sources. In Nepal, vulture deaths have been caused by the ingestion of
diclofenac, an anti-inflammatory medication used to treat cattle on whose carcasses the vultures feed. The
goal of vulture restaurants is to provide a safe source of food to combat these threats in areas where
vultures are known to roost.
• Statements 2 and statement 3 are not correct: The first vulture restaurant was built in South Africa in
1966. Vulture restaurants operate in a number of countries, including Nepal, India, Cambodia, South
Africa, Swaziland, and Spain. The Phansad Wildlife Sanctuary in Maharashtra is playing a part in the
species’ rescue by opening India's first-ever vulture restaurant. In this new restaurant, the birds are fed
their traditional meals of dead cattle, sheep or goats, guaranteed to be diclofenac-free. The vulture’s meal
is laid out on the stony ground in the middle of a clearing, right next to a water trough. The vultures have
also been provided with large trees to alight on.

Q 34.D
• Swami Vivekananda spread Ramakrishna’s message and tried to reconcile it to the needs of contemporary
Indian society. He emerged as the preacher of neo-Hinduism. Certain spiritual experiences of
Ramakrishna, the teachings of the Upanishads and the Gita and the examples of the Buddha and Jesus are
the basis of Vivekananda’s message to the world about human values. He subscribed to the Vedanta
which he considered a fully rational system with a superior approach. Hence statement 1 is correct.
• Vivekananda condemned the caste system and the current Hindu emphasis on rituals, ceremonies
and superstitions, and urged the people to imbibe the spirit of liberty, equality and free thinking. Hence
statement 2 is correct.
• In 1896, Vivekananda founded the Ramakrishna Mission to carry on humanitarian relief and social
work. The Mission had many branches in different parts of the country and carried on social service by
opening schools, hospitals and dispensaries etc. The motto of the Ramakrishna Mission is personal
salvation. Hence statement 3 is correct.
• Hence option (d) is the correct answer.

Q 35.D
• One of the major reasons for the revolt was socio-religious policies of the British which included racial
superiority and discrimination, missionary activities and deliberate efforts to subvert the religious beliefs
of the people in India.
• Socio-religious conditions prevailing in pre-revolt times:
o Indian society was divided on the basis of religion, language, and castes. The Hindu society was
divided into four varnas and multiple sub-castes. The Hindu society was over-sensitive to the ideas of
purity, and severe restrictions were imposed by caste rules

13 www.visionias.in ©Vision IAS


https://telegram.me/pdf4exams/ https://telegram.me/pdf4exams/
o Religion was considered as the most sacrosanct identity among Indians. By and large, there were
many orthodox principles governing religious practices in India. Indians were assumed to have been a
deeply conservative people whose traditions and ways of life had been disregarded by their British
rulers. Reforms, new laws, new technology, even Christianity, had been forced upon them.
• Statement 1 is correct: The conservative religious and social sentiments of many people were hurt by
some of the humanitarian measures which government took on the advice of Indian reformers such as
abolition of practice of Sati, permission of Widow remarriage, opening of Western education to girls. It
was considered as an interference in their faith and values. With regard to the the female education it was
practically unknown and there was no public institution for this purpose. There was a superstitious idea
that a girl taught to read and write would soon afte rmarrying become a widow
• Statement 2 is correct: Religious sentiments were also hurt by the official policy of taxing the lands
belonging to temples and mosques which had been exempted from taxation by previous Indian rulers.
• Statement 3 is not correct: Rediscovery of India's past by Europeans scholars, gave a pyschological
boost to educated Indians later and it has nothing to do with 1857 revolt.

Q 36.D
• The Moplah rebellions happened in Malabar region of South India, which is located in present day
Kerala. Hence, Statement 2 is not correct.
• The series of these uprisings started in 1830s and reached at its most intense form during Khilafat
agitation and Non-Cooperation Movement during 1921. Hence, Statement 3 is not correct.
• Moplahs were mainly Muslim agriculturalists who were mainly of the ranks of landless laborers,
cultivating tenants, fishermen and petty traders. The landlords were mainly high caste Hindus. Hence,
Statement 1 is not correct.
• They were not only directed against British but also the Hindu Landlords.
• In the traditional Malabar land system, the Jenmis held land by birthright and were mostly high caste
Hindus, and let it out to others for cultivation. The land was given by the ruling raja to Namboodiri
Brahmins whose obligation was to look after the temple and related institutions, and to
the chieftains (mostly Nayars), who provided martial aid when needed.
• Traditionally, the net produce of the land was shared equally between the three. But during the reign of
Haider Ali and Tipu Sultan, Namboodiri Brahmins and Nayar Chiefs fled and the subsequent
vacuum was filled by the Moplahs. The conflict arose when after Malabar’s cession to the British in
1792 and the return of the exiled Namboodiri Brahmins and Nayars, the government re-established and
acknowledged their landlord rights.
• The British by recognizing the Jenmis as the absolute owners of the land gave them the right to
evict the tenants at will. This reduced the other two to the status of tenants and leaseholders. The courts
and the law officers sided with the Jenmis. Once the Jenmi landlords, who had the backing of the revenue
officials, the law court and the police started tightening their hold and demands on the subordinate classes,
the Moplah peasantry rose up in revolt.
• The first outbreak occurred in 1836 and during the period of 1834-54 there were 22 uprisings and had
messianic overtones. The second phase of the revolt was recorded in 1882-85, while another spate of
outburst in 1876 was also there.
• The Moplah movement reached its culmination during 1921. During this time, the Moplah movement
took to communal swing. Further, When the Khilafat and Non-Cooperation movement stood for non-
violence and also the struggle for independence, the Moplah took to violence as a method of agitation.

Q 37.D
• The capitalist class’ growth had the features:
o The Indian capitalist class grew from about the mid-19th century with largely an independent capital
base and not as junior partners of foreign capital or as compradors.
o The capitalist class, on the whole, was not tied up in a subservient position with pro-imperialist feudal
interests either economically or politically.
o In the period 1914-1947, the capitalist class grew rapidly, increasing its strength and self-confidence.
This was achieved primarily through import substitution; by edging out or encroaching upon areas
of European domination, and by establishing almost exclusive control over new areas thus
accounting for the bulk of the new investments made since the 1920s.

14 www.visionias.in ©Vision IAS


https://telegram.me/pdf4exams/ https://telegram.me/pdf4exams/
o Close to independence, the indigenous enterprise had already cornered seventy-two to seventy-three
percent of the domestic market and over eighty percent of the deposits in the organized banking
sector.
o The growth of the capitalists was achieved in spite of and in opposition to colonialism — by
waging a constant struggle against colonialism and colonial interests, i.e., by wrenching space from
colonialism itself.
• Hence both statements are not correct.

Q 38.C
• Statement 1 is not correct; Aurangzeb deputed Raja Jai Singh of Amber, who was one of the most
trusted advisers of Aurangzeb, to deal with Shivaji.
• Unlike Shaista Khan, Jai Singh did not underestimate the Marathas rather he made careful diplomatic and
military preparations.
• Jai Singh planned to strike at the heart of Shivaji's territories i.e. fort Purandar where Shivaji had lodged
his family and his treasure.
• In 1665, Jai Singh besieged Purandar (1665), beating off all Maratha attempts to relieve it. With the fall
of the fort at sight, and no relief likely from any quarter, Shivaji opened negotiations with Jai Singh.
• Statement 2 is not correct; After hard bargaining with Shivaji, the following terms we agreed upon
(Treaty of Purandar) −
o Out of 35 forts held by Shivaji, 23 forts were surrendered to the Mughals; Remaining 12 forts were
left with Shivaji on condition of service and loyalty to the Mughal throne.
o Territory worth four lakhs of huns a year in the Bijapuri Konkan, which Shivaji had already held,
was granted to him.
o The Bijapur territory worth of five lakhs of huns a year in the uplands (Balaghat), which Shivaji
had conquered, was also granted to him. In return for these, Shivaji was to pay 40 lakhs huns in
instalments to the Mughals.
o Shivaji asked them to be excused from personal service. Hence, a mansab of 5,000 was granted to
his minor son, Sambhaji.
o Shivaji promised, however, to join personally in any Mughal campaign in the Deccan but there
was no clause for accepting Mughal Suzerainty.

Q 39.A
• Social Service League:
o It was founded in 1911 by Narayan Malhar Joshi, a leading member of the Servants of India
Society.
o The objective of the league was “to collect and study social facts and discuss social problems with a
view to forming public opinion on questions of social service” and to secure for the masses a better
quality of life and work. The League opened a number of day and night schools, libraries, dispensaries
and started boys’ clubs and scouts corps.
o Joshi was intimately associated with the league's working till 1955.
• All India Trade Union Federation:
o It was founded by Narayan Malhar Joshi in 1931.
o V.V. Giri was its first president.
o By 1929, All India Trade Union Congress(AITUC) was the only central trade union. In 1930, there
was a split in the AITUC on account of ideological differences which prompted the creation of
AITUF under the leadership of Shri N.M. Joshi.
o With the formation of AITUF, the strength of the AITUC was reduced to 21 unions with a
membership of 94,000 only. Subsequently, In 1931, there was another rift in the AITUC due to the
fundamental differences between the communist and the left wing unionists. The communists led by
B.T. Ranadive and S.V. Deshpande formed a new organisation named Red Trade Union Congress
(RTUC).
o The AITUF and the railway unions amalgamated with the National Federation of Labour (NFL) under
the new name National Trade Union Federation (NTUF) in 1933.
• Ahmedabad Textile Labour Association (TLA):
o It was founded in 1918 , by Mahatma Gandhi along with Anasuya Sarabhai and Shankerlal
Banker.
15 www.visionias.in ©Vision IAS
https://telegram.me/pdf4exams/ https://telegram.me/pdf4exams/
o It is also known as Majoor Mahajan Sangh.
o With 14,000 workers on its rolls, it was one of the largest single trade unions of the time.
o In 1917, textile workers in Ahmedabad were demanding a 50% hike in plague bonus. After a long
battle based on non-violence, the mill owners finally relented and the victory led to the creation of the
Majoor Mahajan Sangh.
• Girni Kamgar Union:
o It was founded in 1928 under the leadership of Shripad Amrit Dange.
o Muzaffar Ahmed, PC Joshi and Sohan Singh Joshi were the other famous trade union leaders and
communists of that time which influenced its formation.
o It superseded the 'Giri Kamgar Mahamandal' which was founded by A.A. Alve and G.R. Kasle in
Mumbai.
o Girni Kamgar Mahamandal was split and the communists formed their own union, the Girni Kamgar
Union during the general strike of 1928.
o It enabled them to dominate trade union movement. On 30 October 1928, the Girni Kamgar Union
had a membership of 324; by the end of strikes in 1928 and 1929, it boasted to 54000 members.
o S.A. Dange was the general secretary of the Girni Kamgar Union during the strikes.
o Marathi journal, Kranti, was the official organ of the Girni Kamgar Union.

Q 40.A
• Arya Mahila Samaj
o Pandita Ramabai Saraswati founded the Arya Mahila Samaj to serve the cause of women.
o It was started on November 30, 1882 with the aim of empowering and educating each woman for
leading a dignified life. The inaugural meeting was attended by 150 women. On the first day, 16
women members were enrolled. It imagined ideal woman as an efficient housewife, entering the
public world to help during emergencies such as famines, floods and plagues.
• All India Women’s Conference
o The All India Women’s Conference (AIWC), founded by Margaret Cousins in 1927, was perhaps
the first women’s organisation with an egalitarian approach. Its first conference was held at Ferguson
College, Pune. Important founding members included Maharani Chimnabai Gaekwad, Rani Sahiba of
Sangli, Sarojini Naidu, Kamla Devi Chattopadhyaya and Lady Dorab Tata.
Its objectives were to work for a society based on principles of social justice, integrity, equal rights
and opportunities; and to secure for every human being, the essentials of life, not determined by
accident of birth or sex but by planned social distribution.
• Bharat Stree Mahamandal
o In 1910, Sarla Devi Chaudhurani convened the first meeting of the Bharat Stree Mahamandal in
Allahabad. Considered as the first major Indian women’s organisation set up by a woman, its
objectives included promotion of education for women, abolition of the purdah system and
improvement in the socio-economic and political status of woman all over India. Sarla Devi believed
that the man working for women’s upliftment lived ‘under the shade of Manu’.
• Hence, option (a) is the correct answer.

Q 41.A
• The International Migration Stock 2019 report is released by the Population Division of the UN
Department of Economic and Social Affairs (DESA). It provides the latest estimates of the number of
international migrants by age, sex, and origin for all countries and areas of the world. According to the
report,
• The number of international migrants in the world had reached an estimated 272 million in 2019 — 51
million more than in 2010 and the global number of international migrants has grown faster than the
world’s population. Hence statement 1 is correct.
• In 2019, regionally, Europe hosts the largest number of international migrants (82 million),
followed by Northern America (59 million) and Northern Africa and Western Asia (49 million). At
the country level, the United States of America hosts the largest number of international migrants (51
million), equal to about 19 percent of the world’s total. while Saudi Arabia (13 million), Russia (12
million), England (10 million), France (8 million) and Italy (6 million) are also holding the large numbers
of migrants. Hence statement 2 is not correct.

16 www.visionias.in ©Vision IAS


https://telegram.me/pdf4exams/ https://telegram.me/pdf4exams/
• One-third of all international migrants originate from only ten countries, with India as the lead
country of origin, accounting for about 18 million persons living abroad. Migrants from Mexico
constituted the second largest “diaspora” (12 million), followed by China (11 million), the Russian
Federation (10 million) and the Syrian Arab Republic (8 million). Hence statement 3 is not correct.
• Forced displacements across international borders continue to rise. Between 2010 and 2017, the global
number of refugees and asylum seekers increased by about 13 million. Women comprise slightly less than
half of all international migrants in 2019. The share of women and girls in the global number of
international migrants fell slightly, from 49 percent in 2000 to 48 percent in 2019.In terms of age, one out
of every seven international migrants is below the age of 20 years. Three out of every four international
migrants are of working age (20-64 years).

Q 42.C
• On November 1, 1858, a grand Darbar was held at Allahabad. Here Lord Canning sent forth the royal
proclamation which announced that the queen had assumed the government of India. This proclamation
declared the future policy of the British Rule in India.
• It pledged that the British desire no extensions of their present territorial possessions. That is, the
policy of annexation of the princely states and expansion of the empire was to be abandoned. Hence,
Statement 1 is correct.
• All treaties and agreements made with the Indian native princes under the authority of the East
India company did not cease to exist but were there to stay and accepted by the crown. British shall
respect the rights, dignity, and honor of native princes as their own. Also, the princes were also expected
to keep their end of the deal. Hence, Statement 2 is not correct.
• The British Government ordered its servants in India not to interfere in the religious affairs of the Indians.
In framing and administering the law in India, due regard was to be shown to the customs, ancient rites
and usages of the Indians. Hence statement 3 is correct.

Q 43.C
• The moderate politicians never visualized a clinical separation from the British empire; what they wanted
was only limited self-government within the imperial framework. Some of their demands for reforms
were:
• Constitutional reforms:
o Complete separation of the executive and the judiciary. They made this demand to protect Indians
from arbitrary acts by the police and the bureaucracy. Hence statement 1 is correct.
o Right to appeal to the Standing Committee of the House of Commons against the Government
of India. Hence statement 3 is correct.
o Repeal of the arms act the abolition of the India Council which prevented the secretary of state from
initiating liberal policies in India.
o Broaden Indian participation in legislatures through an expansion of the central and provincial
legislatures.
o New councils for North-Western Provinces and Punjab and two Indian members in the Viceroy's
Executive Council.
o The budget should be referred to the legislature, which should have the right to discuss and vote on it
and also the right of interpellation.
o Demand for the abolition of salt tax
• Economic reforms:
o Demand for the extension of the Permanent Settlement to Ryotwari and Mahalwari areas. It was
because the British Indian Association of the landowners maintained a cordial relationship with the
Congress for the first few initial years and remained its major source of finance. About 18.99 per cent
of the delegates who attended the Congress sessions between 1892 and 1909 were landlords. The
Congress, therefore, could not dispense with landed aristocrats and could not consequently take a
logical stand on peasant questions. They demanded an extension of the Permanent Settlement
only in the interest of the zamindars and opposed the cadastral survey in 1893-94, though it was
meant to protect the peasants from the manipulations of the zamindars and their intriguing amlas. The
small pro-tenant lobby within the Congress-led by R.C. Dutt was soon outmaneuvered, as their
opposition in 1898 to the pro-zamindar amendment to the Bengal Tenancy Act of 1885 put them in a
difficult situation. Hence, option 2 is not correct.

17 www.visionias.in ©Vision IAS


https://telegram.me/pdf4exams/ https://telegram.me/pdf4exams/
o Reallocation of military charges
o Protectionist policy to protect Indian industries
o Encouragement of cottage industries and handicrafts
• Administrative reforms:
o Indianisation of the civil services.
o extension of trial by jury
o complaint against over-assessment of land revenue
• All these demands represented a plea for racial equality and a concern for civil rights and also perhaps
reflected a concern for the lower orders, though of a very limited nature.

Q 44.C
• Poligars were the offshoots of the Nayankara system prevalent in the Vijaynagar Administration. The
Poligars were quite similar to the Rajputs of North India and were given land in exchange for military
service when called upon. However, their influence and power increased beyond the traditional lines and
they often acted as sovereigns, even to the extent of extracting taxes from the people. Hence, Statement 1
is correct.
• As the company’s government wanted to augment its own sources of revenues, it sought to control the
Poligars.
• In September 1799, in the first Poligar War, the poligars of Tirunelveli District rose up in open rebellion.
A column of Company troops was speedily deployed against them, while dire warnings were issued to
poligars in other parts of the south not to join the rebellion. Kattabomma Nayak of Panchalamkurichi was
considered as the main leader of the rebellion. Subramania Pillai and Soundra Pandian Nayak were other
important rebel leaders.
• The Second Poligar war of 1800-01, given the magnitude of participation, is also known as the
“South Indian Rebellion”. It was directed by a confederacy consisting of Marudu Pandian of Sivaganga,
Gopala Nayak of Dundigal, Kerala Verma of Malabar and Krishnappa Nayak and Dhoondaji of Mysore.
The rebellion broke out when a band of Poligar armies bombed the combined forces of the poligars. The
suppression was followed by the signing of the Carnatic Treaty on July 31, 1801, whereby the British
assumed direct control over Tamilnadu.
• The Poligar system, which had flourished for two and half centuries, came to a violent end and the
company introduced the Zamindari settlement in its place. Hence, Statement 2 is correct.
• The Poligar Rebellions happened from 1799-1805.

Q 45.D
• Lord Ripon appointed the first Indian Education Commission on 3rd February 1882. Sir Willium Hunter
(a member of viceroy’s Executive Council) was appointed as the chairman of the commission. The
commission was popularly known as Hunter Commission after the name of its chairman. Besides the
chairman, the commission consisted of 20 other members. There was a good representation of
missionaries and Indians in the commission. Among the Indian members were Sayed Mahmud, Bhudev
Mukherjee, Anand Mohan Bose and K.T.Telang.
• The commission was appointed with the following aims:
o To enquire into the manner in which effect had been given to the principles of the Despatch of 1854.
o To assess the position of primary education in India and to suggest measures for its reform.
o To enquire into the position of the State institutions and their importance.
o To evaluate the work of missionaries in the field of education.
o To enquire into Government attitude towards private enterprise.
• The Hunter Commission recommended for both primary and secondary education in India has far-
reaching effect. It recommended the following:
o Hunter Commission made valuable contribution to the development mass education by recognising
primary education as education of the masses.
o Use of mother tongue as the medium of instruction at primary level was also an important
recommendation of the commission. Hence statement 1 is correct.
o Following the method adopted in England, the commission recommended that the control of
primary education should be handed over to District and Municipal Boards. Hence statement 2
is correct.

18 www.visionias.in ©Vision IAS


o Realising the importance of indigenous schools the commission recommended for giving grants to
encourage these schools.
o To improve the quality of teachers, the commission recommended establishment of Normal schools
for the training of teachers.
o The Commission was very liberal in matters of management of the primary schools. Managers should
be given freedom in selecting text books, School hours and holidays should be adjusted according to
local needs.
o The Commission was in favour of maintaining a fund by the District and Municipal Boards
exclusively for maintenance of primary schools. The Commission recommended more practical
curriculum by inclusion of practical subjects in the curriculum of primary schools.
• Statement 3 is not correct: The Hunter commission completely restricted itself to primary and secondary
education. It did not recommend any such move of bringing universities under the state's control.

Q 46.D
• The founder of the autonomous kingdom of Awadh was Saadat Khan Burhan-ul-Mulk who was
appointed Governor of Awadh in 1722. Before his death in 1739, he had become virtually independent
and had made the province a hereditary possession. He was succeeded by his nephew Safdar Jang, who
was simultaneously appointed the wazir of the empire in 1748 and granted in addition the province of
Allahabad.
• Safdar Jang gave a long period of peace to the people of Awadh and Allahabad before his death in 1754.
He suppressed rebellious zamindars, won over others and made an alliance with the Maratha sardars
so that his dominion was saved from their incursions. He was able to win the loyalty of Rajput
chieftains and shaifyizadas.
• He carried on warfare against the Rohelas and the Bangash Pathans. In his war against the Bangash
Pathans in 1750-51, he secured Maratha military help by paying a daily allowance of Rs 25,000 and Jat
support by paying Rs 15,000 a day.
• Later, he entered into an agreement with the Peshwa by which the Peshwa was to help the Mughal
empire against Ahmad Shah Abdali and to protect it from such internal rebels as the Indian
Pathans and the Rajput rajas. In return the Peshwa was to be paid Rs 50 lakhs, granted the chauth of the
Punjab, Sindh, and several districts of northern India, and made the Governor of Ajmer and Agra. The
agreement failed, however, as the Peshwa went over to Safdar Jang’s enemies at Delhi who promised him
the governorship of Awadh and Allahabad.
• Safdar Jang also organised an equitable system of justice. He too adopted a policy of impartiality in the
employment of Hindus and Muslims. The highest post in his government was held by a Hindu, Maharaja
Nawab Rai. The prolonged period of peace and of economic prosperity of the nobles under the
government of the Nawabs resulted in time in the growth of a distinct Lucknow culture around the Awadh
court.
• Lucknow, for long an important city of Awadh, and the seat of the Awadh Nawabs after 1775, soon
rivalled Delhi in its patronage of the arts and literature. It also developed as an important centre of
handicrafts. Crafts and culture also percolated to towns under the patronage of local chieftains and
zamindars.
• Safdar Jang maintained a very high standard of personal morality. All his life he was devoted to his
only wife. As a matter of fact all the founders of the three autonomous kingdoms of Hyderabad, Bengal
and Awadh, namely Nizam-ul-Mulk, Murshid Quli Khan and Alivardi Khan, and Saadat Khan and Safdar
Jang, were men of high personal morality. Nearly all of them led austere and simple lives. Their lives
give lie to the belief that all the leading nobles of the eighteenth century led extravagant and luxurious
lives. It was only in their public and political dealings that they resorted to fraud, intrigue and treachery.
• Hence, all the given statements are not correct.

Q 47.D
• Murshid Quli Khan effected economies in the administration and reorganized the finances of Bengal by
transferring large parts of jagir lands into khalisah lands by carrying out a fresh revenue settlement, and by
introducing the system of revenue-farming. He recruited revenue farmers and officials from local
zamindars and merchant-bankers. He also granted agricultural loans (taccavi) to the poor cultivators
to relieve their distress as well as to enable them to pay land revenue in time. He was thus able to

19 www.visionias.in ©Vision IAS


increase the resources of the Bengal government. But the system of revenue-farming led to increased
economic pressure on the zamindars and peasants.
• Murshid Quli Khan and the succeeding Nawabs gave equal opportunities for employment to Hindus and
Muslims. They filled the highest civil posts and many of the military posts with Bengalis, mostly
Hindus. In choosing revenue farmers Murshid Quli Khan gave preference to local zamindars and
mahajans (money-lenders) who were mainly Hindus. He thus laid the foundations of a new landed
aristocracy in Bengal.
• All the three Nawabs recognised that the expansion of trade benefited the people and the government and,
therefore, gave encouragement to all merchants, Indian and foreign. At the same time they made it a point
to maintain strict control over the foreign trading companies and their servants and prevented them from
abusing their privileges.
• The Bengal Nawabs proved, however, to be short-sighted and negligent in one respect. They did not
firmly put down the increasing tendency of the English East India Company after 1707 to use military
force, or to threaten its use, to get its demands accepted. They failed to see that the English Company was
no mere company of traders but was the representative of the most aggressive and expansionist
colonialism of the time.
• The Nawabs of Bengal neglected to build a strong army and paid a heavy price for it. For example,
the army of Murshid Quli Khan consisted of only 2000 cavalry and 4000 infantry. Alivardi Khan was
constantly troubled by the repeated invasions of the Marathas and, in the end, he had to cede a large part
of Orissa to them. And when, in 1756-67, the English East India Company declared war on Siraj-ud-
Daulah, the successor of Alivardi, the absence of a strong army contributed much to the victory of the
foreigner.
• Hence only statements 1 and 2 are correct.

Q 48.A
• The Indian Council Act of 1892 was an Act of the Parliament that empowered and expanded the
legislative councils in British India.
• The act increased the number of additional or non-official members in the legislative councils as
follows:
• Central Legislative Council: 10 – 16 members Bengal: 20 members Madras: 20 members Bombay: 8
members Oudh: 15 members North Western Province: 15 In 1892, out of 24 members, only 5 were
Indians. Hence, Statement 1 is correct.
• The members were given the right to ask questions on the budget (which was barred in the Indian
Councils Act 1861) or matters of public interest but had to give notice of 6 days for it. However, they
could not ask supplementary questions. Hence, Statement 2 is correct.
• The act made a limited and indirect provision for the use of election in filling up some of the
nonofficial seats both in the Central and provincial legislative councils. The word “election” was,
however, not used in the act. The process was described as 'nomination made on the recommendation of
certain bodies.’ This act laid the foundation of Parliamentary system in India and also was the landmark in
the constitutional development of India. It was the first time the election principles was accepted and
introduced by the act of 1892. However, apart from this, the act fell far short to fulfill National Demands
and did not give anything substantial.
• The power to issue ordinance was conferred on the Viceroy by the Indian Councils Act, 1861. Hence,
statement 3 is not correct.

Q 49.A
• National Intelligence Grid (NATGRID)
o NATGRID is an attached office under the Ministry of Home Affairs.
o It is a robust intelligence gathering mechanism related to banking, immigration, individual
taxpayers, air and train travels.
o It is setup to track any terror suspect and prevent terrorist attacks with realtime data. Hence statement
1 is correct.
o It will have data related to all immigration entry and exit, air flyers, credit card purchases, banking
and financial transactions, individual taxpayers, telecom, and train travelers among others to generate
intelligence inputs.

20 www.visionias.in ©Vision IAS


• Phase I of Project: In the 1st phase, 10 user agencies and 21 service providers will be connected with
NATGRID. In later stages, 950 organizations and then another 1,000 organizations will be connected with
it.
o 10 agencies which will be able to access NATGRID data on a real-time basis are:
▪ Intelligence Bureau (IB)
▪ Research & Analysis Wing (R&AW)
▪ Central Bureau of Investigation (CBI)
▪ Enforcement Directorate (ED)
▪ Directorate of Revenue Intelligence (DRI)
▪ Financial Intelligence Unit (FIU)
▪ Central Board of Direct Taxes (CBDT)
▪ Central Board of Excise and Customs (CBEC)
▪ Directorate General of Central Excise and Intelligence (DGCEI)
▪ Narcotics Control Bureau (NCB)
o Initially, no state agencies will be given direct access to NATGRID data but in case any relevant
information is required, they can approach NATGRID through any of 10 user agencies. There is no
provision for publicly sharing the data. Hence statement 2 is not correct.
• Background
o In the wake of the 2008 Mumbai terror attacks, the lack of real-time data and information was
considered to be one of the major hurdles in detecting terrorist movement across India.
o Thereby in 2010, the Cabinet Committee on Security had given approval to the NATGRID project.
o The project was recently reviewed and taken up for completion.

Q 50.D
• Charles Cornwallis (governor-general, 1786-93) is known as the 'Father of Civil Service in India'.
He introduced Covenanted Civil Services (Higher Civil Services) and Uncovenanted Civil Services
(Lower Civil Services). Cornwallis was the first to bring into existence and organise the civil
services. He tried to check corruption through
o raising the civil servants’ salary,
o strict enforcement of rules against private trade,
o debarring civil servants from taking presents, bribes etc.,
o enforcing promotions through seniority.
• In 1800, Wellesley (governor-general, 1798-1805) set up the Fort William College for the training of
new recruits. In 1806 Wellesley’s college was disapproved by the Court of Directors and instead the East
India College was set up at Haileybury in England to impart two years’ training to the recruits.
• In 1878-79, Lytton introduced the Statutory Civil Service consisting of one-sixth of covenanted
posts to be filled by Indians of high families through nominations by local governments subject to
approval by the secretary of State and the viceroy. But the system failed and was abolished.
• Hence, all the given statements are correct.

Q 51.C
• An Islamic revivalist movement founded by Syed Ahmed of Rai Bareilly, who was inspired by the
teachings of Abdul Wahab (1703-87) of Saudi Arabia and Shah Waliullah of Delhi.
• Syed Ahmed condemned the western influence on Islam and advocated a return to pure Islam and society.
He was declared as Imam (desired leader) and a countrywide organization with and elaborate secret code
for its working under spiritual vice-regents (khalifas) was set up, and Sithana in north-western tribal belt
was chosen as a base for operations. In India, its important centre was at Patna though it had its missions
in Hyderabad, Madras, Bengal, UP and Bombay.
• Since the Dar-Ul-Harb (the land of kafirs) was to be converted into Dar-Ul-Islam (the land of
Islam), a jihad was declared against the Sikh kingdom of Punjab. After the defeat of the Sikh ruler
and incorporation of the Punjab into East India Company’s dominion in 1849, the British dominion in
India became the sole target of the Wahabi’s attacks. A series of military operations by British in 1860s
on the Wahabi base in Sithana and various court cases of sedition on the Wahabis weakened the Wahabi
resistance, although sporadic encounters with the authorities continued into the 1880s and 1890s.
• Hence both the statements are correct.

21 www.visionias.in ©Vision IAS


Q 52.C
• Satya Shodhak was founded in 1873 (Truth-Seeking Society) with Mahatma Jyotiba Phule as its first
president and treasurer and Narayanrao Govindrao Kadalak was elected as its first secretary. Hence,
statement 1 is not correct.
• The main objects of the samaj were -:
o To redeem the Shudras and Ati-Shudras from the influence of Brahmanical scriptures under which the
Brahmin priests fleeced them to make them conscious of their human rights and to liberate them from
mental and religious slavery.
o All men are the children of one God, who is thus their parent
o There is no requirement of an intermediary such as a priest or a preceptor to approach the mother or
father to offer his prayer to god.
• A member at Satya Shodhak Samaj had to take an oath in the name of god Khanderao and declare his
allegiance to British rule. Hence statement 2 is correct. Membership of the samaj was extended to all the
castes including Mahars, Mangs, Jews and Muslims.
• The main objectives of the organisation were to liberate the Shudras and Ati Shudras and to prevent their
exploitation by the upper caste like Brahmans. It opposed idolatry and denounced the chaturvarnya system
(the caste system). It propounded the spread of rational thinking and rejected the need for a Brahman
priestly class as educational and religious leaders. It believed neither in the caste system nor in the basic
four-fold division of society. Hence statement 3 is correct.

Q 53.C
• The three conflicts or wars fought between the British East India Company and the Maratha confederacy
or the Maratha Empire in India are referred to as the great Maratha Wars or the Anglo-Maratha Wars.
• First Maratha War: The main cause of the first Maratha war was the increased interference of the
British in the affairs, both internal and external, of the Marathas and also the struggle for power between
Madhav Rao and Raghunath Rao. Lord Warren Hastings who was the Governor-General of India
from 1773 to 1785, led Britishers in this war. Hence statement 1 is correct.
• Second Maratha War: It was fought between 1803 and 1805. The main cause of the second Maratha war
the defeat of the Peshwa Baji Rao II by the Holkars, one of the prominent Maratha clans, as a result of
which he accepted British protection by signing the Treaty of Bassein in December 1802. The other
Maratha rulers such as Gwalior’s Scindia rulers and the Bhonsle rulers of Nagpur and Berar did not accept
this and they wanted to fight against the British. As a result, the second Anglo-Maratha war broke out.
• The Third Maratha War: The two main causes that led to the third and the final conflict between the
British and the Marathas were the growing desire of the Marathas to get back their lost territories, and
excessive control over Maratha nobles and chiefs by the British. The third War took place in Maharashtra
and its neighboring areas in 1817 and 1818. On 5 November 1817, the Treaty of Gwalior was signed in
which Sindhia became a mere spectator in the war. On 6 January 1818, the Treaty of Mandasor was
signed between Malhar Rao Holkar and the British, in which the Peshwa was dethroned, and this
was followed by the pensioning of the Peshwa. More of his territories were annexed with the British and
the British established their supremacy in India. Hence, statement 3 is correct.
• The Pindari War: Lord Hastings, with the approval of the Court of Directors of the East India Company,
decided to eliminate the Pindaris. The organized campaign by Britishers against Pindaris, known as the
Pindari War, became the Third Anglo-Maratha War. Since the Pindaris gave a portion of their loot to the
Maratha leaders, the Peshwa at Pune, the Bhonsle Raja at Nagpur and the army of the infant Holkar of
Indore each took up arms but were separately defeated.
• The Pindaris were surrounded on all sides by the great army, which converged upon them from Bengal,
the Deccan and Gujarat under the supreme command of Lord Hastings in person. Sindhia was overawed
and forced to sign the treaty of Gwalior, consenting to aid in the extirpation of the Pindaris, whom he had
hitherto protected.
• The Pindaris themselves offered little opposition. Amir Khan, by far their most powerful leader, accepted
the conditions offered to him; he and his descendants became the Nawabs of the state of Tonk in
Rajputana. The rest surrendered or were hunted down, the fate of Chitu, one of the most notorious, is
believed to have perished in a tiger’s den. Hence statement 2 is not correct.

22 www.visionias.in ©Vision IAS


Q 54.B
• The Indian National Congress tried from the very beginning to eliminate the regional differences so that it
could represent itself as a national organization. The first Congress declared that one of its major
objectives would be the "development and consolidation of those sentiments of national unity".
• The decision to hold the Congress session every year in different parts of the country and to choose
the president from a region other than the one where the session was being held was taken. It was
meant to break the regional barriers and misunderstandings. Hence statement 1 is not
correct and statement 2 is correct.
• It was meant to be organised in the way of a parliament and the sessions were conducted
democratically. The resolutions were introduced for the discussions, voting was done on them and so on.
It was meant to induce a culture of political socialisation among the people. Hence statement 3 is
correct.
• The avowed objective of all these endeavours was to create a forum through which the politically
conscious people of different regions of India could unite.

Q 55.A
• Home Charges refer to the expenditure incurred in England by the Secretary of State on behalf of
India. The main constituents were:
o Dividend to the shareholders of the East India Company.
o Interest on Public Debt raised abroad.
o Expenses on India Office establishment in London.
o Pensions and furloughs payments of British officers in the Civil and Military departments in
India.
o Payments to the British war office.
o Store purchases in England.
• Whereas, Economic drain mainly consists of:
o All the Home Charges
o Interests and profits on Foreign Capital Investments
o Payments with regard to banking, insurance and shipping services in India.
• Hence option (a) is the correct answer.

Q 56.C
• The Maxwell-Broomfield commission was set up by the British government to enquire into the
increase in land revenue by the government in the Bardoli Satyagraha. It was set up in the wake of
Bardoli Satyagraha. Hence option (c) is the correct answer.
• The commission came to the conclusion that the increase had been unjustified, and reduced the
enhancement to 6.03 %. In addition, the peasants were returned their confiscated land.
• Bardoli Satyagraha
o It was to protest against the increase in land revenue. It was by Vallabhbhai Patel at Bardoli, Gujarat.
Under the leadership of Patel, the Bardoli peasants decided to refuse payments of the revised
assessment until the Government appointed an independent tribunal or accepted the current amount as
full payment. Those who opposed the movement had to face a social boycott. The outcome of the
satyagraha was British Government setup Maxwell-Broomfield commission, reduced land Revenue to
6.03% and returned confiscated land back to farmers. In this struggle, Vallabhbhai Patel got the title
of “Sardar” by local farmers of Bardoli.

Q 57.A
• In the Bombay Deccan region, the British had introduced the Ryotwari settlement as the system of
land revenue. Under this system, the revenue of land was fixed on a yearly basis. In the Ryotwari system,
the agreement was between the government and the ryot (cultivator) directly. The revenue was fixed
according to the soil-type and the paying capacity of the farmer. However, the revenues were so high
that farmers found it extremely difficult to pay their dues. Any failure in the rains would deteriorate the
situation. To pay their revenues farmers generally took loans from moneylenders. Once the loans were
taken, the farmers found it impossible to repay them since the interest rates were steep. Peasant
indebtedness became a serious problem in rural areas. In 1861, civil war broke out in the USA. The USA
was the largest supplier of cotton in Britain. Once the civil war broke out, the demand for cotton from

23 www.visionias.in ©Vision IAS


India became high and this led to a surge in cotton cultivation in India and there was a period of
‘boom’ then. However, once the war in America ended, cotton demand sunk and this affected the
farmers adversely. The moneylenders, who during the time of the civil war were generous with their
loans, once again refused the farmers' loans. This infuriated the farmers because they were completely
dependent on the moneylenders, who were insensitive to their plight. Hence, option (a) is not correct.
• The movement adopted a variety of methods in its course. When the peasants of the village failed to
convince the local moneylender to not act on a court decree and pull down a peasant’s house, they
organized a complete social boycott of the ‘outsider’ moneylenders to compel them to accept their
demands in a peaceful manner. They refused to buy from their shops. No peasant would cultivate their
fields. The bullotedars (village servants) - barbers, washermen, carpenters, ironsmiths, shoemakers and
others would not serve them. Hence, option (b) is correct.
• The Government acted with speed and soon succeeded in repressing the movement. The active phase of
the movement lasted about three weeks, though stray incidents occurred for another month or two. There
was an absence of anti-colonial consciousness. Hence, the colonial regime extended the peasants the
protection against the moneylenders through the Deccan Agriculturists’ Relief Act of 1879. Hence option
(c) is correct.
• It received immense support from the nationalist intelligentsia of Maharashtra. The Poona Sarvajanik
Sabha, led by Justice Ranade, had organized a successful campaign among the peasants, as well as at
Poona and Bombay against the land revenue settlement. Hence, option (d) is correct.

Q 58.D
• Association of World Election Bodies (A-WEB)
o It is the largest association of Election Management Bodies (EMBs) worldwide.
o It was established on October 14, 2013, in Song-do, South Korea with a Permanent secretariat in
Seoul.
• Composition
o 115 EMBs as Members from 106 countries. Hence statement 1 is not correct.
▪ 24 EMBs from Asia, 37 from Africa, 31 from America, 17 from Europe & 6 from Oceania
presently are members of A-WEB.
o 20 Regional Associations/Organisations as Associate Members.
• Aims and objectives
o It aims to foster efficiency and effectiveness in conducting free, fair, transparent and participative
elections worldwide.
o A-WEB organizes the Election Management Capacity Building Program for election officials to
strengthen the election management capacity, and ultimately to manage their elections in a more
effective and professional manner. Hence statement 2 is not correct.
• Organization
o The Chairperson of A-WEB serves as the head of the association for a term of two years and presides
over the General Assembly and the Executive Board meeting.
o The Chairperson shall be the head of the election management body that hosts the General Assembly.
• Recent Happenings
o Election Commission is hosting the 4th General Assembly of the Association of World Election
Bodies (A-WEB) at Bengaluru.
o India will take over as A-WEB’s Chair for the 2019-21 term.

Q 59.B
• The British India government passed a series of acts to improve the social conditions in India. Some of the
major ones include -:
o Age of Consent Act, 1891
▪ The Age of Consent Act, 1891, also known as Act X of 1891, was legislation enacted in British
India which raised the age of consent for sexual intercourse for all girls, married or unmarried,
from ten to twelve years in all jurisdictions, its violation subject to criminal prosecution as rape.
▪ It was enacted during the tenure of Lord Lansdowne (1888-1894) as the Governor-General.
▪ Hence, pair 1 is not correctly matched.
o Hindu Widows' Remarriage Act, 1856

24 www.visionias.in ©Vision IAS


▪ It legalised the remarriage of Hindu widows. It was passed owing to the efforts of social reformer
Ishwar Chandra Vidyasagar. It was passed during the tenure of Lord Dalhousie (1848-56).
▪ Hence, pair 2 is correctly matched.
o Female Infanticide Prevention Act, 1870
▪ The Female Infanticide Prevention Act, 1870, implemented by the British Government in the
latter part of the nineteenth century was the first statute that aimed to deter people from murdering
female infants by penalizing the action and laying down specific punishments for the same.
▪ It was enacted during the tenure of Lord Mayo (1869-1872) as the Governor-General.
▪ Hence, pair 3 is not correctly matched.

Q 60.D
• Mahalwari System:
o A modified version of the zamindari settlement, introduced in the Gangetic valley, the North-West
Provinces, parts of Central India, and Punjab, was known as the Mahalwari System.
o The revenue settlement was to be made village by village or estate (mahal) by the estate with
landlords or heads of families who collectively claimed to be the landlords of the village or the
estate.
o In Punjab, a modified Mahalwari System known as the village system was introduced.
o In Mahalwari areas also, the land revenue was periodically revised. Hence all the statements are
correct.

Q 61.D
• Rangpur uprising is considered to be the first formidable peasant uprising against the rule of the East India
Company.
• Rangpur uprising happened in opposition to the exploitation caused by the introduction of
Izaradari system by Warren Hastings. The uprising happened in 1783 while Permanent settlement was
introduced in 1793. Hence, Statement 1 is not correct.
• Both Hindu and Muslim peasants fought side by side in the movement and there were no communal
clashes during the rebellion. Hence, Statement 2 is not correct.
• Bakshi Jagbandhu was the leader of the great Paika rebellion of 1817 and not Rangpur rebellion.
Hence, Statement 3 is not correct.
• The period of Warren Hastings was marked by several experiments in land settlement. Under the Ijaradari
system land was farmed out for one to five years to ijaradar. The ijaradar tend to squeeze out as much
money as possible during the period for which the land was taken by him. The burden of revenue was
placed at the highest on the zamindars. The failure to pay revenue would deprive the zamindar of his
estate. Alongside the revenue, other illegal demands made the total demand so high that the ryots were
unable to pay it. So, as remedial measures, the ryots and the zamindars combined to rebel against the
ijaradar, in this case, Devi Singh, the ijaradar of Rangpur and Dinajpur.

Q 62.A
• In 1905 Aurobindo Ghosh's Bhawani Mandir (Temple of the Holy Mother) was banned along
with Vartaman Rananiti (The Technique of Modern Fighting) and Mukti Kon Pathe (Which Way
Freedom?) which he and his brother Barindra Ghose published. The latter contained articles
from Jugantar calling for an armed uprising.
• Brahm Bandhav Upadhyay's Sandhya (it was the most outspoken newspaper). It propagated anti-
British ideas among the people during the Swadeshi movement.
• Hence all the options are correct.

Q 63.B
• Before the organised efforts towards labour reforms began during the Swadeshi Movement, there were
some early attempts to improve the condition of the workers.
• In Bengal, Sasipada Banerjea, a Brahmo Social reformer, set up a Workingmen’s Club in 1870 and
brought out a monthly journal called Bharat Sramjeebi (Indian Labour), with the primary idea of
educating the workers. Hence, Statement 1 is correct.
• In Bombay, Narayan Meghajee Lokhanday brought out an Anglo-Marathi weekly called
Deenbandhu (Friend of the Poor) in 1880. Hence, Statement 2 is correct.
25 www.visionias.in ©Vision IAS
• Bombay Mill and Millhands’ Association was also formed in 1890 by N.M. Lokhanday, not by G.S.
Agarkar. Hence, Statement 3 is not correct. Lokhanday held meetings of workers and in one instance
sent a memorial signed by 5,500 mill workers, to the Bombay Factory Commission, putting forward some
minimum workers’ demands.
• All these efforts were admittedly of a philanthropic nature and did not represent the beginnings of an
organized working class movement. Moreover, these philanthropists did not belong to the mainstream of
the contemporary national movement.
• G.S. Agarkar who was a radical thinker, contributed in publishing articles in Mahratta and
supported the workers’ cause and asked the mill owners to make concessions to them.
• In 1878, Sorabjee Shapoorji Bengalee tried unsuccessfully to introduce a Bill in the Bombay Legislative
Council to limit the working hours for labour.

Q 64.B
• The first session of the Indian National Congress was attended by 72 delegates and presided over
by Womesh Chandra Bonnerjee. Hereafter, the Congress met every year in December, in a different
part of the country each time.
• Some of the great presidents of the Congress during this early phase were Dadabhai Naoroji (thrice
president), Badruddin Tyabji, Pherozshah Mehta, P. Anandacharlu, Surendranath Banerjea,
Romesh Chandra Dutt, Ananda Mohan Bose and Gopal Krishna Gokhale. Badruddin Tyabji was
the first muslim president of Indian National congress.
• Bal Gangadhar Tilak and M.G.Ranade never became presidents of Indian National Congress.
• Hence option b is the correct answer.

Q 65.A
• The Congress had organised a National Conference on Education in 1937 in Wardha. In the light of
the resolutions passed there, Zakir Hussain Committee formulated a detailed national scheme for basic
education. The main principle behind this Wardha scheme of basic education was 'learning through
activity'. Hence statement 2 is correct.
• It was based on Gandhiji's ideas published in a series of articles in the weekly 'Harijan'. Gandhiji
thought that western education had created a gulf between the educated few and the masses and had also
made the educated elite ineffective. Hence statement 1 is correct.
• Following Wardha conference, a committee under Dr Zakir Hussain was appointed to formulate the
scheme of the basic education. The aim of the basic education was to develop the qualities of the ideal
citizenship and more aspect should be given to the Indian culture than the literacy. Also, - There was no
place for English in the curriculum. There was no place for religious education in this scheme. The
scheme centred around ‘ manual productive work’ which might cover the remuneration of the teachers. It
envisaged a seven year course through the mother tongue of the students.
• There was not much development on the ideas of basic education laid down by Wardha scheme
because of the start of the Second World War and the resignation of Congress ministries in 1939.
Hence, statement 3 is not correct.

Q 66.C
• Haidar Ali died in 1782 in the course of the second Anglo-Mysore War and was succeeded by his son
Tipu.
• Tipu Sultan ruled Mysore till his death in 1799. His desire to change with the times was symbolized in the
introduction of a new calendar, a new system of coinage, and new scales of weights and measures.
• He showed a keen interest in the French Revolution. He planted a ‘Tree of Liberty’ at Srirangapatam and
he became a member of a Jacobin Club.
• Statement 1 is correct: He made some attempts to introduce modern industries in India by
importing foreign workmen as experts and by extending state support to many industries. He sent
emissaries to France, Turkey, Iran and Pegu Myanmar to develop foreign trade. He also traded with
China.He even tried to set up a trading company on the pattern of European companies and thus sought to
imitate their commercial practices. He tried to promote trade with Russia and Arabia by setting up state
trading institutions in the port towns.
• Statement 2 is correct: When the British occupied Mysore after defeating and killing Tipu in 1799, they
were surprised to find that the Mysore peasant was much more prosperous than the peasant in British

26 www.visionias.in ©Vision IAS


occupied Madras. Sir John Shore, Governor-General from 1793 to 1798, wrote later that “the peasantry
of his dominions are protected and their labour encouraged and rewarded”. Another British observer
wrote of Tipu’s Mysore as “well cultivated, populous with industrious inhabitants, cities newly founded
and commerce extending”.
• He tried to do away with the custom of giving jagirs, and thus increase state income. He also made an
attempt to reduce the hereditary possessions of the poligars and to eliminate the intermediaries between
the state and the cultivator.
• Tipu Sultan fixed one third of the crop as land revenue: on dry lands and it was collected in
money. Waste lands were rent-free in the first year, rented at 1/4th in the second year in the succeeding
years to the usual amount. Under his contemporary, the Nizam of Hyderabad (1792 A.D. to 1800 A.D.)
the revenue was reduced due to different causes, including the severe famines which visited all western
districts during the years 1792 and 1793 A.D.
• Though not free from contemporary economic backwardness, Mysore flourished economically under
Haidar Ali and Tipu, especially when seen in contrast with its immediate past or with the rest of the
country.

Q 67.B
• Marsiya poetry, which holds special significance for Shia Muslims, is a form of literary expression
that is dedicated to describing the persona of Imam Hussain, a revered figure in the Islamic world,
and the hardships he and his kin underwent during the historic Battle of Karbala. Marsiya is typically
read in the month of Muharram, which ended on September 10 this year. Hence statement 1 is not
correct.
• The word Marsiya means elegy, meaning a poem which is a lament for the dead. In Urdu literature,
Marsiya is written principally in praise of Imam Hussain, the grandson of the Prophet, and his family
members who died at the Battle of Karbala in the year 680 CE in present-day Iraq.
• The Marsiya tradition first evolved in Delhi and the Deccan but reached its zenith under the
patronage of the Nawabs of Lucknow, who encouraged the art form in the 18th and 19th centuries
around the same time when Mughal power was steadily receding. Its most iconic poets from the
19th century, Mir Anis and Mirza Dabir, made a profound impact on Marsiya, making six-line
stanzas the preferred form. In the book ‘Reliving Karbala: Martyrdom in South Asian Memory’, author
Syed Akbar Hyder describes Marsiya as a form of poetry which not only touches upon the death of Imam
Hussain and other events but also delves into his ethic (Akhlaaq) of forgiveness and etiquette (Adab) of
compassion. Hence statement 2 is correct.
• Marsiya is also noteworthy for its depiction of events in 7th century Arabia in a manner which could be
relatable to audiences in South Asia, making the genre popular here. For example, its Arab characters are
depicted in the South Asian setting, having habits and customs like elite North Indian families. According
to Hyder, Marsiya is usually sung, and set to Indian Ragas, creating a fusion of music and poetry.

Q 68.B
• The British enacted four new measures between 1908 and 1912 — the Newspapers (Incitement to
Offences) Act and the Criminal Law Amendment Act of 1908, the Press Act of 1910 and the Prevention
of Seditious Meetings Act of 1911. The aim was to curb press freedom and stop the dissemination of
information which could be detrimental to colonial interests.
• Statement 1 is correct because the Press Act of 1910 empowered the local government to demand a
security fee for any ‘offensive content’ against the government. It provided for the suppression of
seditious or objectionable newspapers, books or other documents. Every person keeping a printing press,
shall at the time of making the same, deposit with the Magistrate before whom the declaration is made
security to such an amount, not being less than five hundred or more than two thousand rupees, as the
Magistrate may in each case think fit to require, in money or the equivalent thereof in securities of the
Government of India Nearly 1,000 papers were prosecuted under the Act.
• Statement 2 is correct because it covered-books, newspapers and also every volume part or division of a
volume, and pamphlet, in any language, and every sheet of music, map, chart or plan separately printed or
lithographed. Even any painting, drawing or photograph or other visible representation was covered. The
British were cautious to avoid repetition of the situation when Amrita Bazar Patrika overnight changed
into an English language newspaper in order to avoid the stringent Vernacular Press Act 1878.

27 www.visionias.in ©Vision IAS


• Statement 3 is not correct because the appeals against the orders and actions lied before High Courts.
Also, Every High Court was to frame rules to regulate the procedure in the case of such applications, the
amount of the cost thereof and the execution of orders passed thereon. The Bill also provided for the
provision of application, within two months of the date of declaration of forfeiture, to a special bench of
three Judges of the High Court, on the question of fact whether the matter directed to is or is not of the
prohibited description of the material defined in the Bill. If the High Court finds that it is not of that
description, it must cancel the order of forfeiture.

Q 69.B
• In the Viceroy executive council, there used to be two members representing the military department-
Commander in chief who was the executive head of the army in India. The second was the military
member who advised Governor-General on military matters. Lord Kitchener who came to India in 1902
objected to such dual control on the military. He wanted to abolish the office of the military member who
advised the Governor General on military matters. But Lord Curzon strongly opposed the idea.
• Hence option (b) is correct.

Q 70.A
• Elephant endotheliotropic herpesviruses (EEHV) or Elephantid betaherpesvirus 1 (ElHV-1) is a
type of herpesvirus, which can cause a highly fatal hemorrhagic disease when transmitted to young
Asian elephants. Hence statement 1 is correct.
• In African elephants, related forms of these viruses, which have been identified in wild populations,
are generally benign, occasionally surfacing to cause small growths or lesions. However, some types of
EEHV can cause a highly fatal disease in Asian elephants, which kills up to 80% of severely affected
individuals. Hence statement 2 is not correct.
• The disease can be treated with the rapid application of antiviral drugs, but this has only been effective in
around a third of cases. It is not transmitted to humans. Hence statement 3 is not correct.

Q 71.B
• Statement 1 is not correct: The early congress nationalists were unwilling to take up the question of
labour versus the indigenous employers. Most of the nationalists denied the need for any
Government legislation to regulate working conditions and actively opposed the Factories Act of
1881 and 1891. Similarly, strikes in Indian textiles mills were generally not supported. They were
opposed to factory reforms like the Mining Bill, which proposed to improve the living condition of
women and children and restrict their employment under a certain age.
• Also, they had a differential attitude towards the workers employed in Europeans enterprises and
those employed in Indian enterprises. One major reason was at this time when the anti-imperialist
movement was in its very infancy, the nationalists did not wish to weaken the common struggle against
British rule by creating any divisions within the ranks of the Indian people. Dadabhai Naoroji, in the very
second session of the Indian National Congress (1886), made it clear that the Congress ‘must confine
itself to questions in which the entire nation has direct participation, and it must leave the adjustment of
social reforms and other class questions to class Congresses.
• Further, representation of the commercial classes among its members also prevented Congress
from taking a pro-working class position. At this stage, the nationalists were unwilling to take up the
question of labour versus the indigenous employer. Besides, The nationalists saw the Government
initiative on labour legislation as dictated by British manufacturing interests which, when faced with
growing Indian competition and a shrinking market in India, lobbied for factory legislation in India which
would, for example, by reducing the working hours for labour, reduce the competitive edge enjoyed by
Indian industry. Further, the early nationalists saw rapid industrialisation as the panacea for the
problems of Indian poverty and degradation and were unwilling to countenance any measure which
would impede this process.
• Statement 2 is correct: The scenario completely altered when the question was of Indian labour
employed in British-owned enterprises. Here the nationalists had no hesitation in giving full support to the
workers. The Indian National Congress and the nationalist newspapers began a campaign against
the manner in which the tea plantation workers in Assam were reduced to virtual slavery, with
European planters being given powers, through legislation to arrest, punish and prevent the running away
of labour.

28 www.visionias.in ©Vision IAS


Q 72.C
• Statement 1 is not correct: The Coalition for Disaster Resilient Infrastructure (CDRI) is an initiative of
the Government of India.
• It was launched by Prime Minister of India at the UN Climate Action Summit,2019 in New York.
• It is an international partnership that will support countries- developed and developing- to build climate
and disaster-resilient infrastructure.
• Developed through consultations with more than 35 countries, CDRI envisions enabling a measurable
reduction in infrastructure losses from disasters, including extreme climate events. Established as a
platform for generating and exchanging knowledge, CDRI will conduct country-specific and global
activities.
• CDRI will provide member countries technical support and capacity development, research and
knowledge management, and advocacy and partnerships to facilitate and encourage investment in
disaster-resilient infrastructure systems.
• In its formative stage, CDRI will focus on developing resilience in ecological infrastructure, social
infrastructure with a concerted emphasis on health and education, and economic infrastructure with
special attention to transportation, telecommunications, energy, and water.
• Statement 2 is correct: CDRI aims to help develop disaster resilient technologies and mechanism for
integrating risk reduction in infrastructure financing.
• The coalition aims to have a 3-fold impact of achieving considerable changes in member countries’ policy
frameworks, future infrastructure investments and high reduction in economic losses from climate-related
events and natural disasters across sectors.
• CDRI will uphold the UN Agenda 2030 principle of leaving no one, no place, and no ecosystem behind,
focusing on the most vulnerable regions and populations, while enabling inclusive and deliberative
processes that recognize national and local efforts as primal.
• Statement 3 is correct: The secretariat of the Coalition for Disaster Resilient Infrastructure (CDRI),
supported by United Nations Office for Disaster Risk Reduction (UNDRR), will be established in New
Delhi. It will facilitate knowledge exchange, provide technical support and support capacity building.

Q 73.D
• Recently, Croatia became the 97th country ratified the Basel Ban Amendment and with this, it becomes
international law. It will become a new Article in the Basel Convention and will enter into force in the 97
countries after 90 days — on December 5.
• Statement 1 is not correct: The Basel Ban Amendment ( part of Basal Convention), a global waste
dumping prohibition which was adopted by the parties to the Basel Convention in 1995, to protect human
health and the environment against the adverse effects of hazardous wastes.
• The Ban Amendment provides for the prohibition of exports of all hazardous wastes covered by the
Convention, including electronic wastes, and obsolete ships from 29 wealthiest countries of the
Organization of Economic Cooperation and Development (OECD) to non-OECD countries.
• The Basel Convention in its COP14 held in 2019 also include unsorted, mixed and contaminated plastic
waste under PIC (Prior Informed Consent) procedure and improve the regulation of its transboundary
movement. This is a significant step taken towards addressing plastic pollution which has become a major
environmental concern across the globe.
• Statement 2 is not correct: India is yet to ratify the Basel Amendment Ban. Recently, Croatia ratified it.

Q 74.C
• Chief of Defence Staff (CDS)
o This post was announced by Prime Minister Narendra Modi on the Independence Day Speech, 2019.
o It will integrate the operations of the three forces -- the Indian Army, the Indian Navy and the Indian
Air Force. Once this post is created, it will provide effective leadership at the topmost level of the
three forces.
o Chief of Defence Staff (CDS) is a post that will act as the single-point advisor to the Government
of India. The officer concerned will be in a position to advise on matters related to all the three
services -- Army, Navy and Air Force -- thus making India's armed forces integrated. Hence
statement 1 is correct.
o The Chief of Defence Staff will be a 'first among equals', a fourth four-star officer who will be senior
to the three other service chiefs. The three forces will continue to have their own chiefs. Hence
statement 2 is correct.

29 www.visionias.in ©Vision IAS


▪ The senior-most of the three chiefs functions as the chairman of the Chiefs of Staff Committee.
However, the Chairman of Chiefs of Staff Committee is effectively the first among equals and
does not wield real power like a Chief of Defence Staff would
o The recommendation for Chief of Defence Staff was first made after the 1999 Kargil War by a high-
level committee that was appointed to review the security system in the aftermath of the conflict.

Q 75.D
• A new, curious mineral has been discovered inside a diamond unearthed from a mine in South
Africa. The mineral has been named goldschmidtite, after Victor Moritz Goldschmidt, the Norwegian
scientist acknowledged as the founder of modern geochemistry. It has been described in the journal
American Mineralogist. Hence option (d) is the correct answer.
• Goldschmidtite has an unusual chemical signature for a mineral from Earth’s mantle. While the
mantle is dominated by elements such as magnesium and iron, goldschmidtite has high
concentrations of niobium, potassium and the rare earth elements lanthanum and cerium.
• PhD student Nicole Meyer found a single grain of the mineral in the diamond, unearthed in Koffiefontein,
South Africa. The university described it as dark green and opaque.
• Though the mantle makes up about 80 per cent of the Earth’s volume, very little is known about it.
Reaching the mantle is not easy; it is about 2,900 km thick and no attempt to drill into it has been
successful. Diamonds hold clues as they are found up to 160 km beneath the surface, in the upper mantle.
Diamonds that are unearthed were brought up closer to the surface, probably as a result of violent volcanic
eruptions when the Earth was hotter.

Q 76.A
• On Aurangzeb’s death his three sons fought among themselves for the throne The 65-year old Bahadur
Shah emerged victorious. He was learned, dignified, and able. He followed a policy of compromise and
conciliation, and there was evidence of the reversal of some of the narrow-minded policies and measures
adopted by Aurangzeb. He adopted a more tolerant attitude towards the Hindu chiefs and rajas.
• His policy towards the Maratha sardars (chiefs) was that of half-hearted conciliation. While he granted
them the sardeshmukhi of the Deccan, he failed to grant them the chauth and thus to satisfy them fully. He
also did not recognise Shahu as the rightful Maratha King. He thus let Tara Bai and Shahu fight for
supremacy over the Maratha Kingdom. The result was that Shahu and the Maratha sardars remained
dissatisfied and the Deccan continued to be a prey to disorder. There could be no restoration of peace and
order so long as the Maratha sardars fought one another as well as fought against the Mughal authority.
• Bahadur Shah had tried to conciliate the rebellious Sikhs by making peace with Guru Gobind Singh
and giving him a high mansab (rank). But when, after the death of the Guru, the Sikhs once again raised
the banner of revolt m the Punjab under the leadership of Banda Bahadur, the Emperor decided to take
strong measures and himself led a campaign against the rebels, who soon controlled practically the entire
territory between the Sutlej and the Jamuna, reaching the close neighbourhood of Delhi.
• There was further deterioration in the field of administration in Bahadur Shah’s reign. The position of
state finances worsened as a result of his reckless grants of jagirs and promotions. During his reign
the remnants of the Royal treasure, amounting in 1707 to some 13 crores of rupees, were exhausted.
Bahadur Shah was groping towards a solution of the problems besetting the Empiie. Given time, he might
have revived the Imperial fortunes. Unfortunately, his death in 1712 plunged the Empire once again into
civil war.
• Hence, only statement 1 is correct.

Q 77.A
• Military expenditure was one of the sore points between the Indian nationalists and the British
government. The British Indian army was being used in imperial wars in all parts of the world,
particularly in Africa and Asia. Hence, Statement 1 is correct.
• These wars in Africa and Asia and the Indian frontier wars of the 1890s put a very heavy burden on the
Indian finances. The moderates demanded that this military expenditure should be evenly shared by
the British government. Hence, Statement 3 is not correct.
• Moderates demanded that Indians should be taken into the army as volunteers. However, British
military officers, particularly, Commander-in-chief Roberts abhorred the idea of volunteer service, as they
feared that the Maratha and Bengali volunteers, disaffected and untrustworthy as they were because of

30 www.visionias.in ©Vision IAS


their association with nationalism, would surely find their way into the army and subvert its
integrity. Hence, Statement 2 is not correct.
• All of these demands were rejected.
• Similarly, the demand for appointing Indians in commissioned ranks was rejected, as no European officer
would cherish the thought of being ordered by an Indian commander. The British government agreed to
share only a small fraction of the military expenditure, less than £1 million in all. The higher exchange
rates reduced the amount even further, and so the burden on the Indian finances remained the same.

Q 78.D
• The Great Game - also known as Bolshaya Igra - was an intense rivalry between the British and
Russian Empires in Central Asia, beginning in the nineteenth century and continuing through 1907
wherein Britain sought to influence or control much of Central Asia to buffer the "crown jewel" of
its empire - British India. Hence option (d) is the correct answer.
• The British Lord Ellenborough started "The Great Game" on January 12, 1830, with an edict establishing
a new trade route from India to Bukhara, using Turkey, Persia, and Afghanistan as a buffer against Russia
to prevent it from controlling any ports on the Persian Gulf. Meanwhile, Russia wanted to establish a
neutral zone in Afghanistan allowing for their use of crucial trade routes.
• This resulted in a series of unsuccessful wars for the British to control Afghanistan, Bukhara, and Turkey.
The British lost at all four wars - the First Anglo-Saxon War (1838), the First Anglo-Sikh War (1843), the
Second Anglo-Sikh War (1848) and the Second Anglo-Afghan War (1878) - resulting in Russia taking
control of several Khanates including Bukhara.
• Although Britain's attempts to conquer Afghanistan ended in humiliation, the independent nation held as a
buffer between Russia and India. In Tibet, Britain established control for just two years after the
Younghusband Expedition of 1903 to 1904, before being displaced by Qin China. The Chinese emperor
fell just seven years later, allowing Tibet to rule itself once more.
• The Great Game officially ended with the Anglo-Russian Convention of 1907, which divided Persia into a
Russian-controlled northern zone, a nominally independent central zone, and a British-controlled southern
zone. The Convention also specified a borderline between the two empires running from the eastern point
of Persia to Afghanistan and declared Afghanistan an official protectorate of Britain.

Q 79.B
• The most important challenge to the decaying Mughal power came from the Maratha Kingdom which was
the most powerful of the succession states. Shahu, the grandson of Shivaji, had been a prisoner in the
hands of Aurangzeb since 1689. Shahu was released in 1707 after Aurangzeb’s death. Very soon a civil
war broke out between Shahu at Satara and his aunt Tara Bai at Kolhapur who had carried out an anti-
Mughal struggle since 1700 in the name of her son Shivaji II after the death of her husband Raja Ram.
• Arising from the conflict between Shahu and his rival at Kolhapur, a new system of Maratha
government was evolved under the leadership of Balaji Vishwanath, the Peshwa of King Shahu.
With this change began the second period—the period of Peshwa domination—in Maratha history in
which the Maratha state was transformed into an empire.
• Balaji Vishwanath gradually consolidated Shahu’s hold and his own over Maratha sardars and over most
of Maharashtra except for the region south of Kolhapur where Raja Ram’s descendants ruled. Already the
system of watans and saranjams (jagirs) had made the Maratha sardars strong, autonomous, and jealous of
central power. They now began to establish their control in the distant lands of the Mughal empire where
they gradually settled down as more or less autonomous chiefs. Thus the conquests of the Marathas
outside their original kingdom were not made by a central army directly controlled by the Maratha
king or the Peshwa but by sardars with their own private armies.
• The Maratha rulers failed to encourage science and technology or to take much interest in trade and
industry. Their revenue system was similar to that of the Mughals as also was their administration. Like
the Mughals, the Maratha rulers were also mainly interested in raising revenue from the helpless
peasantry. For example, they too collected nearly half of the agricultural produce as tax. Unlike the
Mughals, they failed even to give sound administration to the people outside Maharashtra. The only way
the Marathas could have stood up to the rising British power was to have transformed their state into a
modern state. This they failed to do.
• Hence, only statements 1 and 2 are correct.

31 www.visionias.in ©Vision IAS


Q 80.D
• Reforms under Warren Hastings (1772-1785): District Diwani Adalats were established in districts to
try civil disputes. District Fauzdari Adalats were set up to try criminal disputes and were placed under an
Indian officer assisted by qazis and muftis.
• Reforms under William Bentinck (1828-1833): The four Circuit Courts were abolished and their
functions transferred to collectors under the supervision of the commissioner of revenue and circuit. Sadar
Diwani Adalat and a Sadar Nizamat Adalat were set up at Allahabad for the convenience of the people of
Upper Provinces. Till now, Persian was the official language in courts. Now, the suitor had the option to
use Persian or a vernacular language, while in the Supreme Court English language replaced Persian.
• Lord Ripon was associated with the passage of Ilbert bill (1883) according to which the Indian
judges could try a European accused.
• Hence all three were associated with judcial reforms.

Q 81.C
• The world’s first floating nuclear reactor Akademik Lomonosov recently completed its epic 5,000
km Arctic voyage in 22 days, despite environmentalists warning of serious risks to the
region. Loaded with nuclear fuel, it set sail from Murmansk and docked at its permanent location in
Pevek, in Russia’s Far East.
• Akademik Lomonosov was named after the 18th-century Russian scientist Mikhail Lomonosov. It is
developed by Rosatom nuclear agency. The Akademik Lomonosov is scheduled to start operating by this
year-end once connected to the local electricity grid. Once operational, it will become the world’s
northernmost nuclear installation.
• At Pevek, it will operate as part of a floating nuclear power plant. The station houses two 35-megawatt
reactors, more in line with the power of nuclear-powered ice breakers boasting around 1,000 MW
capacity. Once commissioned, it will also become the maiden operational nuclear power plant based on
small modular reactors (SMRs) technology; and a ‘working prototype’ for a reliable source of low-carbon
energy supply in remote areas.
• The Akademik Lomonosov is expected to provide energy for around 1,00,000 people and power oil
platforms as Russia develops extraction of natural resources in a mineral-rich area whose eastern tip is a
few kilometres from Alaska.

Q 82.D
• There was a keen desire for establishing an all India organization. A political conference was held in
Calcutta in 1883. At this conference, Surendra Bath Bannerje referred to the need for an all India
organization. Subsequently, the idea of forming an all India organization was discussed in a private
meeting of seventeen persons after the Theosophical Convention held at Madras in 1884. Allan Octavian
Hume took initiative in organizing an all India organization.It was granted permission by the Viceroy,
Marquis of Dufferin. The first conference of the Congress was to be held at Poona. It was however not
held due to the spread of cholera in Poona. Hence statement 3 is not correct.
• The first meeting of the Indian National Congress in 1885 was attended by seventy-two non-official
Indian representatives. Looking at their regional distribution, thirty-eight came from Bombay
Presidency, twenty-one from Madras, but only four from Bengal, as the Indian Association had
convened its own national conference in Calcutta almost at the same time and the Bengal leaders were
told of the Bombay conference only at the very last moment. Geographically, within the overall
ascendancy of the presidencies, Bengal was gradually slipping from its leadership position, which was
being taken over by Bombay, surging ahead of all other regions in those years. Hence statement 1 is not
correct.
• Apart from the presidencies, seven representatives came from the four principal towns of North-Western
Provinces and Awadh and one each from the three towns of Punjab.
• In their social composition, the Congress from the very beginning suffered from some important
weaknesses, the most significant of them being an uneven representation and total exclusion of the
non-elite groups of Indian society. It was in other words, despite lofty claims, a gathering of
professionals, some landlords, and businessmen, representing primarily the three presidencies of British
India. In their social composition too, the members of the early Congress belonged predominantly to the
high caste Hindu communities and this pattern continued unchanged for more than two decades of its
existence. Hence statement 2 is not correct.

32 www.visionias.in ©Vision IAS


Q 83.D
• Among the numerous tribal revolts, the Santhal 'hool' or uprising was the most massive one.
• The Santhals lived scattered in various districts of eastern India. With the introduction of permanent
settlement in Bengal in 1793, the Santhals were employed as labourers with the promise of wages or rent-
free lands. However they were forced to become agricultural surfs, exploited at will. Tribal lands were
leased out to non-Santhal zamindars and moneylenders. To this was added the oppression of the local
police and the European officer engaged in railroad construction. This penetration of outsiders by the
Santhals completely destroyed their familiar world and forced them into action to take possession of
their lost territory. These outsiders were called 'Dikus' by the Santhals. Hence, option (b) is not
correct.
• The first rebellion of messianic character erupted in 1854 under Bir Singh of Sasan in Lachimpur.
• In July 1855, when their ultimatum to the zamindars and the government went unheeded, several thousand
Santhals, armed with bows and arrows, started an open insurrection against the unholy trinity of their
oppressors-the zamindars, the mahajans and the government. The insurrection spread rapidly and in a
wide region between Bhagalpur and Rajmahal, the Company's rule virtually collapsed, spreading panic in
government circles. At this stage, the Santhal rebels were also being actively helped by the low caste
non-tribal peasants. Hence, option (c) is not correct.
• This invited brutal counter-insurgency measures; the army was mobilised and Santhal villages were burnt
one after another with vengeance. According to one calculation, out of thirty to fifty thousand rebels,
fifteen to twenty thousand were killed before the insurrection was finally suppressed. Therefore, the
modes used in the rebellion as well as the suppression of the movement were highly violent. Hence,
option (a) is not correct.
• Henceforth, the British government became more cautious about them and the Santhal inhabited areas
were constituted into a separate administrative unit, called the Santhal Parganas, which recognised the
distinctiveness of their tribal culture and identity. Hence, option (d) is correct.

Q 84.B
• Ripon's Resolution of 1882: The Government of Ripon desired the provincial governments to apply in
case of local bodies the same principle of financial decentralisation which Lord Mayo's Government had
begun towards them. The main points of the resolution were as follows:
o Development of local bodies advocated to improve the administration and as an instrument of
political and popular education;
o Policy of administrating local affairs through urban and rural local bodies charged with definite duties
and entrusted with suitable sources of revenues;
o Non-officials to be in majority in these bodies, who could be elected if the officials thought that
it was possible to introduce elections. Hence statement 1 is correct.
o Non-officials to act as chairpersons to these bodies. Hence statement 2 is correct.
o Official interference to be reduced to the minimum and to be exercised to revise and check the acts of
local bodies, but not to dictate policies;
o Official executive sanction required in certain cases, such as raising of loans, alienation of
municipal property, imposition of new taxes, undertaking works costing more than a prescribed
sum, framing rules and bye-laws, etc. Hence statement 3 is not correct.
• In pursuance of this resolution many Acts were passed between 1883 and 1885 which greatly altered the
constitution, powers and functions of municipal bodies in India.

Q 85.A
• The UNCCD is one of the three Rio Conventions—along with the UN Framework Convention on Climate
Change (UNFCCC) and the Convention on Biological Diversity (CBD)—and was called for in Agenda
21, the programme of action adopted at the 1992 UN Conference on Environment and Development
(UNCED, or Earth Summit).
• Statement 1 is correct: 14th Conference of Parties (COP14) to UNCCD was first time held in India.
India being the global host for COP 14 will take over the COP Presidency from China for the next two
years till 2021.
• India is privileged to be among the select few countries to have hosted the COP of all three Rio
conventions on climate change, biodiversity and land.

33 www.visionias.in ©Vision IAS


• Through hosting COP 14, India will highlight its leadership in navigating the land management agenda at
the global level. It will also provide a stage to mainstream sustainable land management in the country's
national development policies.
• The theme of the COP was “Restore land, sustain future” with a focus on achieving Land Degradation
Neutrality (LDN) globally, in line with target 15.3 of the SDGs.
• Statement 2 is not correct: The concept of LDN emerged from the UN Conference on Sustainable
Development (Rio+20) in 2012.
• At the twelfth session of the Conference of Parties of the UNCCD held in October 2015 in Ankara,
Turkey, country parties reached a breakthrough agreement to endorse the vision to achieve Land
Degradation Neutrality (LDN) worldwide by 2030 and link the implementation of the Convention to
the SDGs in general, and target 15.3 in particular.
• As per the principle , country establish national voluntary targets for LDN as a mean to sustainably
increase food security, reduce biodiversity losses, and contribute to climate change adaptation and
mitigation.
• The Land Degradation Neutrality Target Setting Project ensures strong Government leadership as well as
active involvement of other stakeholders and sectors.

Q 86.D
• Nilgiri tahr(IUCN-Endangered), the state animal of Tamil Nadu and endemic to Western Ghats, has
witnessed a 27% increase in its population in the last three years. Its sightings in the Mukurthi National
Park have risen from 568 in 2018 to 612 this year. This was the second consecutive year that an increase
in the population of the animal had been recorded in the park, meaning the population of the Nilgiri tahr,
also known as the Nilgiri ibex, has risen by 132 since 2016. There was a decrease in tahr numbers in
2017, when a population of only 438 was recorded, down from 480 in 2016.
• The Mukurthi National Park is located in the western corner of the Nilgiris Plateau west of
Ootacamund hill station in the northwest corner of Tamil Nadu state in the Western Ghats
mountain range of South India. The vegetation, climate and appearance of the Mukurthi National Park
bear a striking resemblance to that of the Himalayas minus the snow. The park is part of the Nilgiri
Biosphere Reserve which is India's first Biosphere Reserve. The climate is extremely cold and the
rainfall is always on the heavy side.
• The Nilgiri Biosphere Reserve is an International Biosphere Reserve in the Western Ghats and Nilgiri
Hills ranges of South India. The Nilgiri Sub-Cluster is a part of the Western Ghats, which was declared a
World Heritage Site by UNESCO in 2012. It includes the Aralam, Mudumalai, Mukurthi, Nagarhole,
Bandipur and Silent Valley national parks, as well as the Wayanad and Sathyamangalam wildlife
sanctuaries. Hence option (d) is the correct answer.

Q 87.D
• By the 18th century, the Portuguese in India lost their commercial influence, though some of them still
carried on trade in their individual capacity and many took to piracy and robbery. The decline of the
Portuguese was brought about by several factors.
• The local advantages gained by the Portuguese in India were reduced with the emergence of powerful
dynasties in Egypt, Persia and North India and the rise of the turbulent Marathas as their immediate
neighbors. (The Marathas captured Salsette and Bassein in 1739 from the Portuguese.)
• The religious policies of the Portuguese, such as the activities of the Jesuits, gave rise to political fears.
Their antagonism for the Muslims apart, the Portuguese policy of conversion to Christianity made Hindus
also resentful.
• Their dishonest trade practices also evoked a strong reaction. The Portuguese earned notoriety as sea
pirates. Their arrogance and violence brought them the animosity of the rulers of small states and the
imperial Mughals as well.
• The earlier monopoly of knowledge of the sea route to India held by the Portuguese could not
remain a secret forever; soon enough the Dutch and the English, who were learning the skills of
ocean navigation, also learned of it. As new trading communities from Europe arrived in India,
there began a fierce rivalry among them. In this struggle, the Portuguese had to give way to the more
powerful and enterprising competitors. The Dutch and the English had greater resources and more
compulsions to expand overseas, and they overcame the Portuguese resistance. One by one, the
Portuguese possessions fell to its opponents.

34 www.visionias.in ©Vision IAS


Q 88.A
• The most important feature of the labour movement during the Swadeshi days was the shift from
agitations and struggles on purely economic questions to the involvement of the worker with the
wider political issues of the day. The labour movement had graduated from relatively unorganized and
spontaneous strikes on economic issues to organized strikes on economic issues with the support of
the nationalists and then on to working-class involvement in wider political movements. Hence,
Statement 1 is correct.
• The national upsurge on the day the partition of Bengal came into effect, included a spurt of
working-class strikes and hartals in Bengal. Workers in several jute mills and jute press factories,
railway coolies and carters, all struck work. Workers numbering 12,000 in the Bum Company shipyard in
Howrah struck work on being refused to leave to attend the Federation Hall meeting called by the Calcutta
Swadeshi leaders. Workers also went on strike when the management objected to their singing Bande
Mataram or tying rakhis on each others’ wrists as a symbol of unity.
• With the decline in the nationalist mass upsurge after 1908, the labour movement also suffered an
eclipse. It was only with the coming of the next nationalist upsurge in the immediate post-World-War I
years that the working-class movement was to regain its strength, though now on a qualitatively higher
plane. Hence, Statement 2 is not correct.
• Beginning with the Home Rule Leagues in 1915 and continuing through the Rowlatt Satyagraha in
1919, the national movement once again reached great heights in the Non-Cooperation and Khilafat
Movement in 1920-22. There occurred a resurgence of working-class activity in the years from 1919
to 1922. The working class now created its own national-level organisation to defend its class rights,
called AITUC(All India Trade Union Congress). It was in this period that the working class also got
involved in the mainstream of nationalist politics to a significant extent. Hence, Statement 3 is not
correct.

Q 89.D
• Recently, the International Astronomical Union (IAU) has named an asteroid, discovered in 2006,
after Indian classical singer Pandit Jasraj.
• The International Astronomical Union (IAU) was founded in 1919. Its mission is to promote and
safeguard the science of astronomy in all its aspects, including research, communication, education and
development, through international cooperation. Hence statement 1 is correct. The IAU has 13675
members.
• The key activity of the IAU is the organization of scientific meetings.
o Every year the IAU sponsors nine international IAU Symposia.
o Definition of fundamental astronomical and physical constants. Hence statement 2 is correct.
o Promotion of educational activities in astronomy and informal discussions on the possibilities for
future international large-scale facilities.
o The IAU serves as the internationally recognized authority for assigning designations to celestial
bodies and surface features on them. Hence statement 3 is correct.
• Related Information:
o Johann Bach, Ludwig van Beethoven and Rabindranath Tagore, also have minor planets named
after them.
o Sangeet Martand Pandit Jasraj (b. 1930) is an exponent of Indian classical vocal music.
o The asteroid, or more formally known as a minor planet, is located between Mars and Jupiter and
was discovered on November 11, 2006, by the Catalina Sky Survey, whose telescopes are based
in Arizona in the United States.
• Naming of Planet-process:
o The privilege of naming a planet is first given to discoverers, who have 10 years to propose a
name.
o The discoverer or team is expected to write a short citation, explaining the reasons for assigning
the name, according to the IAU’s guidelines.
o All names proposed are judged by the 15-member Working Group for Small Body Nomenclature
(CSBN) of the IAU, comprising professional astronomers with research interests in minor planets
and/or comets from around the world.
Q 90.A
• The British conquest led to the deindustrialization of the country and increased dependence of the people
on agriculture. No figures for the earlier period are available but, according to Census Reports, between
1901 and 1941 alone the percentage of population dependent on agriculture increased from 63.7
percent to 70 percent. Hence, statement 1 is correct.
35 www.visionias.in ©Vision IAS
• The ruin of Indian industries, particularly rural artisan industries, proceeded even more rapidly
once the railways were built. The railways enabled British manufactures to reach, and uproot the
traditional industries in the remotest villages of the country. Hence, statement 2 is not correct.
• Most of the modern Indian industries were owned or controlled by British capital. Only in the cotton
textile industry did the Indians have a large share from the beginning, and in the 1930s, the sugar industry
was developed by the Indians.
• Another serious weakness of Indian industrial effort was the almost complete absence of heavy or
capital goods industries, without which there can be no rapid and independent development of industries.
India had no big plants to produce iron and steel or to manufacture machinery. A few petty repair
workshops represented engineering industries and a few iron and brass foundries represented
metallurgical industries. The first steel in India was produced only in 1913. Thus India lacked such basic
industries as steel, metallurgy, machine, chemical, and oil. India also lagged behind in the development of
electric power. Hence, statement 3 is not correct.

Q 91.C
• James Augustus Hickey in 1780 started The Bengal Gazette or Calcutta General Advertiser, the
first newspaper in India. It was seized in 1872 because of its outspoken criticism of the Government.
Later more newspapers/journals came up - The Bengal Journal, The Calcutta Chronicle, The Madras
Courier, The Bombay Herald. Hence, statement 1 is correct.
• Digdarshan was the first vernacular newspaper. It started in April 1818 by the Serampur missionaries
William Carcy, Joshua Marshman and William Ward. They soon started another journal in June of the
same year and named it Samachar Darpan. The first vernacular newspaper published by an Indian was in
Sambad Kaumudi (in Bengali) and Mir’at’l-Akhbar (in the Persian). It was published by the most
important personality of vernacular journalism, Raja RamMohan Roy. Hence, statement 2 is not correct.
• The Associated Press of India (API) was the first news agency in India. It was formed when some of
the English language papers like the Statesman, Englishman and Indian Daily News pooled their
resources together and brought into being the Associated Press of India. Hence statement 3 is correct.

Q 92.A
• Sanyasi rebellion - 1763
o Sanyasi rebellion began in 1763 and lasted until 1800. It was made famous by Bankim Chandra
Chatterjee in his novel Anand Math. The rebellion witnessed the rising up of displaced peasants and
demobilized soldiers of Bengal led by religious monks.
• Travancore rebellion led by Dewan Velu Thampi - 1805
o Velu Thampi, the Dewan of Travancore, rose up in rebellion against the British attempt to remove
him from the Dewanship and the heavy burden imposed on the state through the Subsidiary Alliance
System. In one of the ensuing skirmishes Velu Thampi was injured and died in the forest. However,
even though dead, he was publicly hanged as an example to the fate of those who rose against the
British.
• Kittur uprising led by Chennamma - 1824
o Kittur uprising was led by Rani Chennamma, the Queen of Kittur. She was one of the first Indian
rulers to lead an armed rebellion against the British East India Company in 1824, against the
implementation of the Doctrine of Lapse.
• Second Anglo-Sikh War - 1848-49
o The Second Anglo-Sikh War was a military conflict between the Sikh Empire and the British East
India Company that took place in 1848 and 1849. It resulted in the fall of the Sikh Empire, and the
annexation of the Punjab and what subsequently became the North-West Frontier Province, by the
East India Company.

Q 93.D
• Statement 1 is not correct: Recently “Bombay blood group”, a rare blood type, has been at the centre of
attention in Mumbai’s healthcare scene as demand for the blood type has coincidentally spiked at
hospitals, but supply has been scarce. The rare, Bombay blood group was first discovered in Mumbai
(then Bombay) in 1952 by Dr Y.M. Bhende.
• Each red blood cell has antigen over its surface, which helps determine which group it belongs to. The
Bombay blood group, also called hh, is deficient in expressing antigen H, meaning the RBC has no

36 www.visionias.in ©Vision IAS


antigen H. For instance, in the AB blood group, both antigens A and B are found. A will have A antigens;
B will have B antigens. In hh, there are no A or B antigens.
• Globally, the hh blood type has an incidence of one in four million. It has a higher incidence in South
Asia; in India, one in 7,600 to 10,000 are born with this type. This blood type is more common in South
Asia than anywhere else because of inbreeding and close community marriages. It is genetically
passed. Shared common ancestry among Indians, Sri Lankans, Pakistanis and Bangladeshis has led to
more cases of hh blood phenotype in this region.
• Statement 2 is not correct: To test for hh blood, an Antigen H blood test is required. Often the hh blood
group is confused with the O group. The difference is that the O group has Antigen H, while the hh group
does not. “The individuals with Bombay blood group can only be transfused autologous blood or blood
from individuals of Bombay hh phenotype only which is very rare.” Rejection may occur if they receive
blood from A, B, AB or O blood group. In contrast, hh blood group can donate their blood to ABO blood
types.

Q 94.B
• The Indian Currency Committee or Fowler Committee was a committee appointed by the British in
1898 to examine the currency situation in India. The committee was chaired by Sir Henry Fowler.
• Until 1892, silver was the metal on which Indian currency and coinage had largely been based. The
committee recommended that the official Indian rupee be based on the gold standard and the official
exchange rate of the rupee be established at 15 rupees per British sovereign, or 1 shilling and 4 pence per
rupee. The British Imperial Government accepted the recommendations of the commission in July 1899.
• The action of the government in abandoning silver coinage was driven by the relative decline of the
value of silver against gold, which had led to an accompanying decline of the rupee against gold and
gold-based currencies such as the British sovereign. India had been importing about a quarter of the
annual global production of silver, in part to meet its currency issuing requirements.
• Artificially fixing the exchange rate of Rupee at such a high rate negatively impacted the Indian
capitalists, and this step was resented by the Congress.

Q 95.C
• Kamini Roy
o Born in Basanda Village of Bakerkunj district of erstwhile Bengal Presidency, (now Bangladesh) in
1864, Kamini Roy was the first woman to graduate with honours in British India. Roy took the
entrance exam to Bethune College in 1880 and cleared it. Despite having many obstacles, she
graduated from Bethune College in 1886 and became the first Indian woman honours graduate with a
bachelor’s degree in Sanskrit.
o While in college, she took an active part in the Ilbert Bill agitation. After graduation, she became a
teacher at Bethune and published her first book of poems named “Alo O Chhaya“, in 1889. She was
motivated by social worker Abala Bose, to join the movement for women’s suffrage in Bengal. In
1921, Roy formed the Bangiya Nari Samaj with Kumudini Mitra and Mrinalini Sen to fight for
suffrage and women’s liberation. She then became a member of the Female Labour
Investigation Commission from 1922 to 1923 which worked with the government to oversee the
conditions of women.
o Apart from being a feminist leader she was a great poet, and some of her notable contribution to
Bengali literature are “Pundorik”, “Dwip O Dhup”, “Jibon Pathey”, “Pauraniki”, “Gunjan”,
“Nirmalya”, “Mallya O Nirmalya”, “Ashoka Sangeet”, and “Alo O Chaya”. Roy was president of
Bangla Literature Summit (1930). Hence, option (c) is the correct answer.
• Kadambini Ganguly
o Kadambini Ganguly was one of the first two female graduates from India. She also went on to
become one of the first female doctors of the country who were certified to practise western medicine.
o Kadambini married Dwarakanath Ganguly a Brahmo Samaj reformer.
o She became the first woman to be on the dais at a session of the Indian National Congress.
• Chandramukhi Basu
o She passed her First Arts examination from Dehradun Native Christian School in 1880, then became
one of the first non-Hindu Indian girls to be admitted into Bethune College. Along with Kadambini
Ganguly, she received her Bachelor’s degree in Arts from Bethune College and School in 1883, thus

37 www.visionias.in ©Vision IAS


becoming one of the first woman graduates in India. She went on to become the first woman MA
graduate from Calcutta University in 1884.
o She became the lecturer as well as the Principal at Bethune College
• Kamaladevi Chattopadhyay
o In early 1920s, when she was in London with her Husband, she came to know about the Non-
cooperation Movement and returned back to join the Seva Dal.
o In Seva Dal, she was in charge of the women’s wing and recruited, trained and organized the Sevikas.
o She contested elections for Madras Provincial Legislative Assembly, thus becoming the first woman
to contest election for a legislative seat, although she lost this election.
o During the Salt Satyagraha, she was member of seven member lead team created by Mahatma Gandhi
to prepare salt at the Bombay beach front.
o In 1950s, her focus turned to Indian handicrafts, arts, handlooms and traditional artisans especially the
women artisans. She established a series of “crafts museums” to preserve and protect India’s arts and
crafts; and also instituted the National Awards for Master Craftsmen; Central Cottage Industries
Emporia throughout India.

Q 96.D
• Treaty of Yandabo
o The first war with Burma was fought when the Burmese expansion westwards and occupation of
Arakan and Manipur, and the threat to Assam and the Brahmaputra Valley led to continuous friction
along the ill-defined border between Bengal and Burma, in the opening decades of the nineteenth
century. It was signed during the tenure of Lord Amherst.
o The British expeditionary forces occupied Rangoon in May 1824 and reached within 72 km of the
capital at Ava. Peace was established in 1826 with the Treaty of Yandabo which provided that the
Government of Burma:
▪ pay rupees one crore as war compensation;
▪ cede its coastal provinces of Arakan and Tenasserim;
▪ abandon claims on Assam, Cachar and Jaintia;
▪ recognise Manipur as an independent state;
▪ negotiate a commercial treaty with Britain; and
▪ accept a British resident at Ava, while posting a Burmese envoy at Calcutta.
o Hence pair 1 is not correctly matched.
• Treaty of Sinchula
o The Bhutan war is also known as Duar War and this ended in the defeat of the Bhutanese army. The
peace was brought by the Treaty of Sinchula which was signed in 1865. Bhutan ceded territories in
the Assam Duars and Bengal Duars, as well as around 80,000 kilometers of Dewangiri (Deothang) to
British in return for an annual subsidy of 50,000 rupees. It was signed during the tenure of Sir John
Lawrence.
o Hence pair 2 is not correctly matched.
• Treaty of Gandamak
o It was signed during the tenure of Lord Lytton. This treaty was signed after the Second-Anglo
Afghan War, 1879 provided that:
▪ the Amir conduct his foreign policy with the advice of Government of India;
▪ a permanent British resident be stationed at Kabul
▪ the Government of India gives Amir all support against foreign aggression, and an annual
subsidy.
o Hence, pair 3 is not correctly matched.
• Treaty of Sagauli
o It was signed between the Gurkha chiefs of Nepal and the British Indian government that ended the
Anglo-Nepalese (Gurkha) War (1814–16). By the treaty, Nepal renounced all claim to the disputed
Tarai, or lowland country, and ceded its conquests west of the Kali River and extending to the Sutlej
River.

38 www.visionias.in ©Vision IAS


o Nepal remained independent, but it received a British resident with the status of an ambassador to an
independent country rather than of the controlling agent of the supreme government in an Indian state.
It was signed during the tenure of Francis Rawdon-Hastings, 1st marquess of Hastings.

Q 97.C
• The Governor-General of India, Lord Auckland, paranoid about Russian incursion or influence north of
India, the “Jewel in the Crown of the British Empire”, issued what was known as the Simla Manifesto in
1838. The Simla Manifesto stated that the welfare of India required that the British have on their
western frontier a trustworthy ally.
• The manifesto stated that in order to ensure the welfare of India, the British must have a trustworthy ally
on India's western frontier. While the Britishers argued that their troops were merely supporting Shuja's
(Afghanistan ruler) small army in retaking what was once his throne, the actual reason was to ensure the
establishment of a British-friendly throne. Although the Simla Manifesto stated that British troops would
be withdrawn as soon as Shuja was installed in Kabul, Shuja's rule depended entirely on British arms to
suppress the rebellion and on British funds to buy the support of tribal chiefs. The British denied that they
were invading Afghanistan, instead claiming they were merely supporting its legitimate Shuja government
"against foreign interference and factious opposition."
• Hence option (c) is the correct answer.
• Other famous events which took place in Shimla which should not be confused with are:
o Shimla Deputation (1906)
▪ In 1906 a group of Muslim landed magnates and chieftains organized a deputation of Lord Minto,
the Viceroy, at Simla and pleaded for a few concession to the Muslim community of India.
▪ However, within a few months following the Simla Deputation, a political forum of the
communalist Muslims emerged. Nawab Salimullah of Dacca took initiative in organizing a
conference of the communalist Muslims in which a resolution was adopted in favour of the
foundation of the All India Muslim League.
▪ Accordingly, the All India Muslim League was founded in 1906.
• Shimla Accord (1914)
o The Simla Accord or the Convention was a treaty concerning the status of Tibet negotiated by
representatives of the Republic of China, Tibet and the United Kingdom in Shimla.
o The Accord provided that Tibet would be divided into "Outer Tibet" and "Inner Tibet". Outer Tibet
would remain in the hands of the Tibetan Government at Lhasa under Chinese suzerainty. Inner Tibet
would be under the jurisdiction of the Chinese government.
• Shimla Conference (1945)
o It was a meeting between the Viceroy of India Lord Wavell and the major political leaders of British
India at Simla. It was convened to agree on and approve the Wavell Plan for Indian self-government
which was related to a proposal for reconstituting the Executive Council in consultation with Indian
leaders.
o The conference broke down on the insistence of Jinnah that his party should have an exclusive right to
nominate Muslim members of the Viceroy’s Executive Council. This was something which the
Congress could not concede without repudiating its national composition.

Q 98.D
• The Lancashire millowners were interested in getting the hours of work and employment of women and
children regulated in order to enhance the cost of production of the Indian mills which had become their
competitors.
• A committee was appointed in 1875 to inquire into the conditions of factory work in the country.
Accordingly, the first Factories Act was adopted in 1881. Following this act, a Factory Commission was
appointed in 1885. There was another Factories Act in 1891, 1911 and 1922.
• Factory Act 1881:For the first time British tried to improve the working conditions of laborers.
o The act prohibited employment of children under the age 7. Children between the age 7-12 were
to work for maximum 9 hours.
o Dangerous machines should be fenced properly to avoid accidents.
39 www.visionias.in ©Vision IAS
o The act had a provision of one hour rest during the working period.
o 4 days leave in a month for workers was made compulsory.
o To supervise the implementation of this act,inspectors were appointed.
o This act was applicable only to factories using mechanical powers,employing not less than 100
workers.
• Factory Act-1891:
o In 1890, Indian factory commission appointed and on its report the next factory act of 1891 was
passed.
o Minimum age of children raised from 7 to 9; and between 9 and 14 years, work limit was only
for 8 hours with no work at night.
o For women, employment at night was prohibited.
o For all workers including male, a mid day stoppage and one day rest per week was prescribed.
o The act applied to all factories employing not less than 50 persons
• Factory Act-1911:
o In 1906, the textile factory and labour committee was appointed and on its report the factory act of
1911 was enacted.
o For the first time, hours of work for adult male workers was fixed at 12 hours/day.
o Certification of children's age was required.

Q 99.B
• The first important parliamentary act regarding the Company's affairs was the Regulating Act of 1773.
This Act made changes in the constitution of the Court of Directors of the Company and subjected their
actions to the supervision of the British Government. The Directors were to lay before the Ministry all
correspondence dealing with the civil and military affairs and the revenues of India
• . In India, the Government of Bengal was to be carried on by a Governor-General and his Council who
were given the power to superintend and control the Bombay and Madras Presidencies in matters of war
and peace.
• The Act also provided for the establishment of a Supreme Court at Calcutta to administer justice to
Europeans, their employees, and the citizens of Calcutta.
• The Regulating Act soon broke down in practice. It had not given the British Government effective and
decisive control over the Company. In India it had placed the Governor-General at the mercy of his
Council. Three of the Councillors could combine and outvote the Governor-General on any matter. In
practice, Warren Hastings, the first Governor-General under the Act, and three of his Councillors
quarreled incessantly, often creating deadlocks in the administration. The Governor-General’s control
over the other two Presidencies also proved inadequate in practice.
• Most important of all, the Act had failed to resolve the conflict between the Company and its
opponents in England who were daily growing stronger and more vocal. Moreover, the Company
remained extremely vulnerable to the attacks of its enemies as the administration of its Indian possessions
continued to be corrupt, oppressive, and economically disastrous.
• The defects of the Regulating Act and the exigencies of British politics necessitated the passing in 1784 of
another important act known as Pitt’s India Act. This Act gave the British Government supreme control
over the Company’s affairs and its administration in India. It established six Commissioners for the affairs
of India, popularly known as the Board of Control, including two Cabinet Ministers. The Board of
Control was to guide and control the work of the Court of Directors and the Government of India. In
important and urgent matters it had the power to send direct orders to India through a secret committee of
Directors. The Act placed the Government of India in the hands of the Governor-General and a Council of
three so that if the Governor-General could get the support of even one member, he could have his way.
The Act clearly subordinated the Bombay and Madras Presidencies to Bengal in all questions of war,
diplomacy, and revenues. With this Act began a new phase of the British conquest of India. While the
East India Company became the instrument of British national policy, India was to be made to serve the
interests of all sections of the ruling classes of Britain.
• Hence, only statement 2 is correct.

40 www.visionias.in ©Vision IAS


Q 100.A
• Ministry of Commerce & Industry through Export Credit Guarantee Corporation (ECGC) has introduced
a new Export Credit Insurance Scheme (ECIS) called NIRVIK. Hence statement 2 is not correct.
• NIRVIK aims to make loans easier to access for exporters and also simplifies the lending process. Hence
statement 1 is correct.
• The ECGC provides credit guarantee of up to 60 percent loss, but under Nirvik, the insurance cover
guaranteed will cover up to 90 percent of the principal and interest of loans and will include both pre and
post-shipment credit.
• If there is any loss, then in that case, ECGC will refund 90 percent to the banks, including principal and
interest. Banks will get up to 50 percent within 30 days after the complaint is lodged
• Nirvik is expected to make Indian exports competitive and benefit MSME exporters with tax
reimbursements, reduced insurance costs, and ease of doing business.
• About ECGC
o It is a company wholly-owned by the Government of India and is based in Mumbai, Maharashtra.
o It is under the Ministry of Commerce, it provides export credit insurance support to Indian exporters.
o The Government of India had initially set up Export Risks Insurance Corporation in 1957, and it was
transformed into Export Credit and Guarantee Corporation Limited (ECGC) in 1964 and to Export
Credit Guarantee Corporation of India in 1983.

Copyright © by Vision IAS


All rights are reserved. No part of this document may be reproduced, stored in a retrieval system or transmitted
in any form or by any means, electronic, mechanical, photocopying, recording or otherwise, without prior
permission of Vision IAS.

41 www.visionias.in ©Vision IAS

You might also like